Uworld Respiratory

Ace your homework & exams now with Quizwiz!

52. The nurse cares for a client with a pulmonary embolism. Which of the following clinical manifestations would the nurse anticipate? Select all that apply. A - Chest pain B- Dyspnea C - Bradycardia D - Hypoxemia E - Tachypnea

A - Chest pain B - Dyspnea D - Hypoxemia E - Tachypnea

The nurse provides discharge instructions to a 67-year-old client with chronic bronchitis who was hospitalized for community-acquired pneumonia. Which instructions should be included in the discharge teaching plan? Select all that apply. a. Avoid the use of OTC cough suppressant medicines b. Oral antibiotics are not needed at home as you had IV therapy in the hospital c. Pneumonia vaccination is not needed as you now have lifelong immunity d. Schedule a follow-up with the HCP and CXR e. Use a cool mist humidifier in your bedroom at night f. Use the incentive spirometer at home

Clients should be taught to understand that symptoms of pneumonia (eg, cough, sputum production, shortness of breath, fatigue, and activity intolerance) remain after discharge even though the bacteria are no longer present and will dissipate over a 2-4 week period, depending on current health status and preexisting conditions. Discharge teaching includes the following instructions: Avoid the use of over-the-counter cough suppressant medicines. Unless prescribed by the HCP, cough suppressants are avoided as they impair secretion clearance, especially in clients with chronic bronchitis. Schedule a follow-up with the HCP and chest x-ray. Follow-up is needed at about 2 weeks after completion of antibiotic therapy. X-ray may be needed at a later time in certain high-risk clients to make sure the pneumonia is resolved with no underlying cancer. Use a cool mist humidifier in your bedroom at night. Humidifiers keep mucus membranes moist, maintain effectiveness of the mucociliary escalator, and facilitate expectoration of mucus. A warm bath also loosens the secretions. Continue using the incentive spirometer at home. Deep breathing and coughing promote lung expansion, ventilation, oxygenation, and airway clearance. Drink 1-2 liters of water a day, if not contraindicated, to help thin secretions and facilitate mobilization. Limit caffeine and alcohol as they can dry mucus membranes due to diuretic effects. Notify the HCP of any increase in symptoms (eg, shortness of breath, cough, sputum production, chest pain, fever, confusion). Avoid all tobacco products and second-hand smoke as these irritate the airways and impair mucociliary clearance and oxygenation. Eat a balanced diet, increase activity slowly over about 2 weeks, and take rest periods when needed to help maintain resistance to infection. (Option 2) After a client has IV antibiotic therapy, completing a full course of oral antibiotic therapy is necessary to prevent reoccurrence of disease and antibiotic resistance. (Option 3) Contracting pneumonia does not provide lifelong immunity to the disease. Yearly influenza vaccination and pneumonia vaccination as directed by the HCP are recommended. Educational objective:Discharge instructions for a client recovering from pneumonia focus on proper medication regimen, lung expansion and coughing techniques, activity level, hydration, nutrition, avoidance of tobacco products, reportable manifestations (eg, respiratory distress, chest pain, fever, cough, change in mucus), follow-up care, influenza and pneumonia vaccinations, and respiratory and hand hygiene.

A nurse is completing discharge teaching to the parent of a child who is postoperative following a tonsillectomy. Which finding should be reported as a priority? a. Ear pain b. Frequent swallowing c. Low-grade fever d. Objectionable mouth odor

Tonsillectomy is usually performed as an outpatient procedure. Postoperative bleeding is an uncommon but important complication and it can last up to 2 weeks. It manifests with frequent or continuous swallowing and/or cough from the trickling blood; some clients may also develop restlessness. Discharge teaching includes: Avoid coughing, clearing the throat, or blowing of the nose Limit physical activity Milk products are discouraged due to their coating effect, which can prompt clearing of the throat Oral mouth rinses, gargling, and vigorous tooth brushing should be avoided to prevent irritation (Options 1, 3, and 4) The presence of slight ear pain, a low-grade fever, and objectionable mouth odor are common findings during the first 5-10 days after the procedure. Persistent moderate-to-severe earache, fever, or cough requires further evaluation. Educational objective:Postoperative bleeding after a tonsillectomy is uncommon but can last up to 14 days after surgery. Continuous swallowing, restlessness, and frequent coughing are early indicators of bleeding. To prevent hemorrhage, the client should avoid clearing the throat, blowing the nose, and coughing.

A client is brought to the emergency department following a motor vehicle collision. The client's admission vital signs are blood pressure 70/50 mm Hg, pulse 123/min, and respirations 8/min. The nurse anticipates the results of which diagnostic test to best evaluate the client's oxygenation and ventilation status? a. ABGs b. CXR c. Hematocrit and Hgb levels d. Serum lactate level

a. ABGs Arterial blood gas (ABG) assessment parameters provide objective data about the efficiency of gas exchange in the lungs and effectively evaluate the following: Acid-base balance (pH, HCO3) Oxygenation status (PaO2, partial pressure of oxygen in the arterial blood) Tissue oxygenation (SaO2, percentage of available hemoglobin saturated with oxygen) Ventilation (PaCO2, partial pressure of carbon dioxide in the arterial blood) Respiratory failure can occur when oxygenation is inadequate (hypoxemic failure) and/or when ventilation is inadequate (hypercapnic failure). The adequacy of oxygenation and ventilation in a client with respiratory failure is best evaluated through ABG analysis. (Option 2) Chest x-ray is used to determine structural abnormality (eg, enlarged heart, fractured ribs), presence of air, fluid, infiltrates, lesions, and response to treatment. It does not provide objective data about a client's gas exchange, oxygenation, and ventilation status. (Option 3) Decreased serum hematocrit and hemoglobin levels can affect the carrying capacity and delivery of oxygen to the tissues. They do not provide objective data about a client's gas exchange, oxygenation, and ventilation status. (Option 4) The serum lactate level provides information about anaerobic tissue metabolism (perfusion). It does not provide objective data about a client's gas exchange, oxygenation, and ventilation status. Educational objective:Respiratory failure can occur when oxygenation is inadequate (hypoxemic failure) and/or when ventilation is inadequate (hypercapnic failure). Arterial blood gas analysis provides objective data about the efficiency of gas exchange in the lungs.

The charge nurse of the emergency department (ED) is mentoring a new registered nurse (RN). They are caring for a client who has a chest tube connected to wall suction for a pneumothorax. The client is being transferred from the ED to the telemetry unit. Which action by the new RN would cause the charge nurse to intervene? a. Clamping the chest tube at the insertion site during the transfer b. Disconnecting the suction tubing from the wall suction unit c. Hanging the chest tube collection unit to the underside of the stretcher d. Taping connections between the chest tube and suction tubing.

a. Clamping the chest tube at the insertion site during the transfer Clamping the chest tube during transport is contraindicated. Doing so can cause air to accumulate in the pleural cavity as it has no means of escape. This can lead to the development of a tension pneumothorax, a potentially life-threatening condition. A tension pneumothorax results in compression of the unaffected lung and pressure on the heart and great vessels. As the pressure increases, venous return is decreased and cardiac output falls. (Option 2) The wall suction needs to be temporarily disconnected during transport. It should be reconnected promptly at the destination. (Option 3) The chest tube collection unit should be hung below the level of the chest to promote drainage and keep fluids from re-entering the chest cavity. (Option 4) All connections should be secured with tape to prevent accidental disconnection or air to enter the system. Educational objective:Chest tubes should not be clamped during transport of a client. A clamped chest tube may cause a tension pneumothorax, a potentially life-threatening event.

The nurse is providing discharge instructions on the proper use of prescribed short-acting beta agonist and inhaled corticosteroid metered-dose inhalers to a client with newly diagnosed asthma. Which instructions should the nurse include? Select all that apply. a. Omit the beclomethasone if the albuterol is effective b. Rinse your mouth well after using the beclomethasone inhaler and do not swallow the water c. Take the albuterol inhaler apart and wash it after every use d. use the albuterol inhaler first if needed, then the beclomethasone inhaler e. Use the beclomethasone inhaler first, then the albuterol if needed

b. Rinse your mouth well after using the beclomethasone inhaler and do not swallow the water e. Use the beclomethasone inhaler first, then the albuterol if needed Asthma is a disorder of the lungs characterized by reversible airway hyper-reactivity and chronic inflammation of the airways. Albuterol (Proventil) is a short-acting beta agonist (SABA) administered as a quick-relief, rescue drug to relieve symptoms (eg, wheezing, breathlessness, chest tightness) associated with intermittent or persistent asthma. Beclomethasone (Beconase) is an inhaled corticosteroid (ICS) normally used as a long-term, first-line drug to control chronic airway inflammation. When using an ICS metered-dose inhaler (MDI), small particles of the medication are deposited and can impact the tongue and mouth. Rinsing the mouth and throat well after using the MDI and not swallowing the water are recommended to help prevent a Candida infection (thrush) (white spots on tongue, buccal mucosa, and throat), a common side effect of ICSs. The use of a spacer with the inhaler can also decrease the risk of developing thrush (Option 2). When both MDIs are to be taken at the same time, clients are instructed to take the SABA first to open the airways and then the ICS to provide better delivery of the medication. It is important for the nurse to clarify indications and sequencing as the SABA is a rescue drug taken on an as-needed basis and is not always taken with the ICS (Options 4 and 5). (Option 1) Inhaled corticosteroids (eg, fluticasone, beclomethasone) are not rescue drugs. They are prescribed to be taken on a regular schedule (eg, morning, bedtime) on a long-term basis to prevent exacerbations and should not be omitted even if the SABA is effective. (Option 3) Taking the albuterol (Proventil) inhaler apart, washing the mouthpiece (not canister) under warm running water, and letting it air dry at least 1-2 times a week is recommended. Medication particles can deposit in the mouthpiece and prevent a full dose of medication from being dispensed. Taking the ICS inhaler apart and cleaning it every day is recommended. Educational objective:Proper use of the short-acting beta agonist (SABA) inhaler includes taking it apart and rinsing the mouthpiece with warm water 1-2 times a week. Proper use of the inhaled corticosteroid (ICS) inhaler includes taking it apart and rinsing the mouthpiece with warm water daily and rinsing the mouth and throat after each use to prevent a Candida infection (thrush). When these medications are administered together, the sequence is SABA first to open the airways and ICS second.

A client comes to the emergency department and reports headache, nausea, and shortness of breath after being stranded at home without electricity due to severe winter weather. While collecting a history, which question is most important for the nurse to ask? a. Are you up to date with your annual flu shot and other vaccinations? b. Have you had difficulty eating or drinking in the last few days c. How have you been keeping your house warm during this weather? d. Is there anything that you have found that relieves your symptoms?

c. How have you been keeping your house warm during this weather? Carbon monoxide (CO) is a colorless, odorless gas produced by burning fuel (eg, oil, kerosene, coal, wood) in a poorly ventilated setting. CO toxicity (poisoning) is most often associated with smoke inhalation from structure fires, but is also generated by furnaces/hot water heaters fueled by natural gas or oil, coal or wood stoves, fireplaces, and engine exhaust. Clients with CO toxicity often have nonspecific symptoms, and the diagnosis can be missed. It is important to assess for possible CO exposure to initiate appropriate emergency care and prevent hypoxic neurologic impairment. To help identify elevated CO levels in the home, the nurse can ask about the following: Similar symptoms in other family members, or an illness in an indoor pet that developed at the same time Fuel-burning heating/cooking appliances; risk of CO toxicity increases in the fall and winter due to increased used of heat sources in an enclosed space (Option 3) (Options 1, 2, and 4) It is important to reconcile the client's vaccinations, obtain a nutritional history, and explore the nature of the client's symptoms, but it is essential to rule out the possibility of CO toxicity given the circumstances of this client's illness. Educational objective:Carbon monoxide (CO) toxicity can occur when fuel-burning (eg, wood, coal) stoves or appliances are used in poorly ventilated settings. Clients with CO toxicity may have vague symptoms (eg, headache, dizziness, nausea), so it is important to assess for possible CO exposure to prevent delay of appropriate emergency care.

GOOD The nurse is caring for a client involved in a motor vehicle collision who had a chest tube inserted to evacuate a pneumothorax caused by fractured ribs. Where would the nurse observe an air leak? a. suction control chamber b. water seal chamber c. collection chamber d. Section D

water seal chamber The presence of an air leak is indicated by continuous bubbling of fluid at the base of the water seal chamber. If the client has a known pneumothorax, intermittent bubbling would be expected. Once the lung has re-expanded and the air leak is sealed, the bubbling will cease. The nurse is expected to assess for the presence or absence of an air leak and to determine whether it originates from the client or the chest tube system. (Option 1) Section A is the suction control chamber. Gentle, continuous bubbling indicates that suction is present. (Option 2) Section B is part of the water seal chamber, but an air leak will not be evident in this upper portion. Tidaling of fluid is expected in this portion of the chamber and indicates patency of the tube. (Option 4) Section D is the collection chamber, where drainage from the client will accumulate. The nurse will assess amount and color of the fluid and record these as output. Educational objective:An air leak is indicated by bubbling of fluid in the base of the water seal chamber of a chest tube drainage unit. The client with a known pneumothorax is expected to have an intermittent air leak, with bubbling in the water seal chamber. Continuous bubbling indicates an air leak somewhere in the chest tube system.

A self-employed auto mechanic is diagnosed with carbon monoxide poisoning. Admission vital signs are blood pressure 90/42 mm Hg, pulse 84/min, respirations 24/min, and oxygen saturation 94% on room air. What is the nurse's priority action?

Administer 100% oxygen using a nonrebreather mask with flow rate of 15 L/min - to treat hypoxia and eliminate CO.

A client with a history of degenerative arthritis is being discharged home following an exacerbation of chronic obstructive pulmonary disease. After reviewing the discharge medications, the nurse should educate the client about which topics? Select all that apply. Click on the exhibit button for additional information. Discharge medications Albuterol: 2 puffs every 4-6 hours as needed Prednisone: 40 mg PO daily Naproxen: 220 mg PO twice daily Tiotropium: 1 capsule inhaled daily a. Dryness of the mouth and throat my occur b. Ringing in the ears is an expected, transiet side effect c. The albuterol canister should not be shaken before use d. The health care provider should be notified if stools are black and tarry e. Tiotropium capsules should not be swallowed.

A common side effect of tiotropium (Spiriva) and other anticholinergics (eg, ipratropium, benztropine) is xerostomia (dry mouth) due to the blockade of muscarinic receptors of the salivary glands, which inhibits salivation. Sugar-free candies or gum may be used to alleviate dry mouth and throat (Option 1). Tiotropium capsules should not be swallowed. These capsules are placed inside the inhaler device, and the capsule is pierced, allowing the client to inhale its contents (Option 5). Glucocorticoids (eg, prednisone), when taken in combination with aspirin or nonsteroidal anti-inflammatory drugs (NSAIDs) such as naproxen, can increase the risk of gastrointestinal ulceration and bleeding. The client should report black, tarry stools (ie, melena) to the health care provider as they could indicate gastrointestinal bleeding (Option 4). (Option 2) Tinnitus (ie, ringing in the ears) is an uncommon side effect of NSAID (eg, naproxen) use. Tinnitus is commonly associated with toxicity related to salicylate-containing NSAIDs (eg, aspirin) or aminoglycosides (eg, gentamicin, neomycin, tobramycin); its onset should be reported by a client taking these medications. The medication may need to be discontinued to prevent permanent hearing loss. (Option 3) The albuterol canister should be shaken prior to inhalation to ensure appropriate medication delivery. Educational objective:The nurse should teach the client taking glucocorticoids with aspirin or nonsteroidal anti-inflammatory drugs about the risk for gastrointestinal bleeding or ulceration. Xerostomia is a common side effect of anticholinergic drugs that can be alleviated with sugar-free candies or gum. Tiotropium capsules should not be swallowed.

The emergency nurse admits a semiconscious client with periorbital bruising and severe tongue edema after a laceration sustained in an unwitnessed tonic-clonic seizure. The health care provider prescribes a nasopharyngeal airway to maintain airway patency. Which initial action by the nurse is appropriate? a. Contact the HCP and clarify the prescription b. Ensure correct placement after insertion by auscultating the lungs c. Select an appropriate size by measuring from nose tip to earlobe d. Verify that the client has a history of bleeding disorders or aspirin use.

A nasopharyngeal airway (NPA) is a tube-like device used to maintain upper airway patency. NPAs are frequently used in alert or semiconscious clients, as they are less likely to cause gagging, and in clients with oral trauma or maxillofacial surgery. NPAs should never be inserted in clients who may have head trauma (eg, facial or basilar skull fractures), such as might occur during an unwitnessed seizure. NPAs inserted in clients with skull fractures may be malpositioned into underlying tissues/structures (eg, brain). Therefore, the nurse should immediately clarify prescriptions for NPAs in clients with head trauma (Option 1). An NPA may be inserted after imaging (eg, CT scan) rules out fracture. (Option 2) Once skull fracture is ruled out and an NPA is inserted, the nurse verifies appropriate airway placement by auscultating the lungs. (Option 3) Inappropriate NPA size increases the risk for airway obstruction, sinus blockage, and infection. To select an appropriate size, the nurse measures from the tip of the client's nose to the earlobe and selects a diameter smaller than the naris. (Option 4) Bleeding disorders and use of anticoagulant or antiplatelet medication (eg, aspirin) are relative contraindications to NPA insertion, as these increase the risk of bleeding. However, skull fracture must be excluded prior to placement. Educational objective:A nasopharyngeal airway (NPA) is a tube-like device used to maintain upper airway patency. NPAs should not be inserted in clients with suspected head trauma until skull fracture can be excluded as there is a risk for unintentional malpositioning into underlying tissue/structures (eg, brain).

A student nurse initiates oxygen with a nonrebreather mask for a client with acute respiratory distress. While reassessing the client, the RN notices the reservoir bag is fully deflating on inspiration. What immediate action does the RN take to correct the problem? a. Elevates the HOB b. Increases the oxygen flow c. Opens both flutter valves (ports) on the mask d. Tightens the face mask straps

A nonrebreather mask is an oxygen delivery device used in a medical emergency. It consists of a face mask with an attached reservoir bag and a one-way valve between the bag and mask that prevents exhaled air from entering the bag and diluting the oxygen concentration. The liter flow must be high enough (up to 15 L/min) to keep the reservoir bag at least 2/3 inflated during inhalation and to prevent the buildup of carbon dioxide in the bag. (Option 1) Elevating the head of the bed allows for maximum chest expansion and promotes oxygenation. It does not inflate the reservoir bag on inhalation or affect the proper operation of the rebreather mask. (Option 3) Ports (exhalation valves) are located on each side of the mask and are covered with rubber discs that act as flutter valves. The valves close on inhalation to prevent entry of room air and open on exhalation to prevent reinhalation of exhaled air. The ports should be occluded when initiating the device to fill the reservoir with oxygen. (Option 4) The nonrebreather mask can deliver high concentrations of oxygen if the mask is secured tightly to the face with the head strap to minimize leaks. Tightness of the mask does not affect the filling or deflating of the reservoir bag. Educational objective:A nonrebreather mask is an oxygen delivery device used in a medical emergency. It can deliver up to 95%-100% oxygen concentration if properly maintained during use. Proper care of the device includes monitoring the reservoir bag to assure continual inflation during inhalation; monitoring the 2 exhalation (flutter) valves that cover the ports on each side of the mask; and keeping the mask secured to the face by adjusting the tightness of the head strap to minimize leaks.

The nurse is assisting a client with asthma perform a peak flow meter measurement. Place the instructions for measuring peak expiratory flow using a peak flow meter in the correct order. All options must be used. a. Exhale as quickly and completely as possible and note the reading on the scale b. Position the indicator on the flow meter scale to the lowest value and assume an upright position c. Record the highest of the three measured values in the peak flow log d. Repeat the procedure 2 more times with a 5-10 second rest period between exhalations. e. Inhale deeply, place mouthpiece in mouth, and use the lips to create the seal 1. __________ 2. __________ 3. __________ 4. __________ 5. __________

A peak flow meter is a handheld device that measures the client's ability to push air out of the lungs. Measurements from a peak flow meter often guide the client's use of respiratory medications and the need to schedule an appointment with a health care provider. To obtain the most accurate readings to help guide, maintain, and evaluate treatment in clients with asthma, the procedure is performed in the following order: Before each use, slide the indicator on the numbered scale on the flow meter to 0 (or the lowest value), and stand or sit as upright as possible (Option 3). Inhale deeply, place the mouthpiece in the mouth, and close the lips tightly around the mouthpiece to form a seal (Option 2). Exhale as quickly and completely as possible and note the reading on the numbered scale (Option 1). Repeat the procedure 2 more times, with a 5- to 10-second rest period between exhalations (Option 5). Record the highest reading (ie, personal best) in the peak flow log (Option 4). Educational objective:When performing peak flow measurements, set the indicator to the lowest value; assume an upright position; inhale deeply; place the mouthpiece in the mouth and form a seal with the lips; exhale quickly and completely; note the value; repeat 2 more times; and then record the highest value in the peak flow log.

The nurse is caring for a client receiving mechanical ventilation via tracheostomy 2 weeks following a tracheotomy. The nurse enters the client's room to address a ventilator alarm and notes the tracheostomy tube dislodged and lying on the client's chest. Which action by the nurse is appropriate? Click on the exhibit button for additional information. VS: HR - 132 RR - 40/min O2 sat% - 80% a. Apply a non rebreather face mask with 100% oxygen b. Apply dry, sterile gauze over the stoma and secure with tape c. Insert a new tracheostomy tube using the bedside obturator d. Insert a sterile catheter into the stoma and suction the airway

A tracheostomy tube, an artificial airway inserted into the trachea through the neck, may be secured with sutures or tracheostomy ties. Accidental dislodgment of a tracheostomy tube is a medical emergency often resulting in respiratory distress from closure of the stoma and airway loss. If accidental dislodgment of mature tracheostomies (ie, >7 days after insertion) occurs where the tract is well formed, the nurse should attempt to open the airway by inserting a curved hemostat to maintain stoma patency and insert a new tracheostomy tube with an obturator (Option 3). (Option 1) Application of supplemental oxygen via nonrebreather face mask may not resolve respiratory distress because air can escape from the stoma. (Option 2) Covering the stoma with a sterile, occlusive dressing (eg, petroleum gauze, foam tape) and ventilating the lungs with a bag-valve mask over the nose/mouth may be necessary if the tube cannot be reinserted or the stoma is immature. Dry gauze is porous and does not adequately seal the stoma for ventilation. (Option 4) Tracheal suctioning may be necessary once the airway is resecured. However, suctioning prior to establishing an airway does not improve ventilation and may further reduce the oxygen supply. Educational objective:Accidental dislodgment of a tracheostomy tube is a medical emergency. With a mature tracheostomy, an attempt to insert a new tracheostomy tube with the bedside obturator is indicated. If a tube cannot be reinserted, the stoma is covered with a sterile, occlusive dressing. Ventilation is provided with a bag-valve mask over the nose/mouth.

A client is scheduled for allergy skin testing to identify asthmatic triggers. Which medications should the nurse instruct the client to withhold before the test to ensure accurate results? Select all that apply. a. Acetaminophen b. Albuterol c. Diphenhydramine d. Enalapril e. Loratidine

Allergy skin testing involves introducing common environmental and food allergens (ie, antigens) into the skin surface and then observing the site for an allergic reaction (eg, formation of a wheal, erythema). Several different antigens, as well as positive and negative controls, are usually tested at the same time for accuracy. To ensure an accurate result, the client should avoid antihistamines (eg, diphenhydramine [Benadryl], loratadine [Claritin], promethazine [Phenergan]) for up to 2 weeks prior to the test (Options 3 and 5). Antihistamines block mast cell release of histamines that are responsible for allergic symptoms. Systemic corticosteroids, used to treat the inflammatory component of asthma, may also affect the accuracy of allergy skin testing; therefore, the use of these medications is assessed by the health care provider. (Option 1) Acetaminophen does not have antihistamine properties and will not interfere with allergy skin testing. (Option 2) Albuterol, an inhaled short-acting beta adrenergic agonist, will not interfere with allergy skin testing results and should not be discontinued, as it is necessary to ensure client safety during acute asthma exacerbations. (Option 4) Enalapril, an ACE inhibitor, is used to treat high blood pressure and heart failure and will not impact the results of allergy skin testing. Educational objective:Allergy skin testing involves introducing common allergens (ie, antigens) into the skin surface and then observing the site for an allergic reaction (eg, formation of a wheal, erythema). Clients should avoid antihistamines as these drugs can prevent accurate results.

A client with Alzheimer disease is found slumped over the lunch tray on the bedside table, coughing violently with emesis visible in the back of the throat. The client has a pulse of 135/min, respirations 32/min, and oxygen saturation 84%. The client also has circumoral cyanosis and decreased level of consciousness. Place the nurse's actions while awaiting the arrival of the rapid response team in priority order. All options must be used. a. Assess lung sounds b. Notify the HCP c. Administer 100% oxygen by non rebreather mask d. Place client in a High Fowler's position e. Perform oropharyngeal suctioning.

Alzheimer disease is a neurological condition that increases the risk for aspiration and aspiration pneumonia, a common cause of death in clients with swallowing dysfunction. The nurse activates a rapid response because the client is in acute respiratory distress. While waiting for the team, the nurse should implement the following actions in order: Place in high Fowler's position - quickly maximizes ability to expand lungs, promotes oxygenation, and helps to decrease risk of further aspiration Perform oropharyngeal suctioning - the priority is clearing the airway after the client has been placed in a position that prevents further aspiration Administer 100% oxygen by nonrebreather mask - corrects hypoxemia/hypoxia once the airway has been cleared to allow passage of oxygen. The nurse has already gathered focused assessment data and determined the need for emergent oxygen delivery (eg, tachycardia, tachypnea, hypoxia, cyanosis, decreased level of consciousness). Assess lung sounds - determines air movement and presence of adventitious sounds (eg, crackles, wheezing, stridor) that can indicate obstruction, secretions, atelectasis, or fluid. This assessment is performed once emergency measures are in place (eg, oxygen) and the client has been stabilized. Notify the primary HCP - to report the situation and assessment data To provide more efficient care, any of these tasks can be delegated to a second RN. Educational objective:While waiting for the rapid response team to respond to an adult client with acute respiratory distress, the nurse implements the following actions: positioning; suctioning to clear the airway, administering high-concentration oxygen; assessing lung sounds; and notifying the HCP.

The nurse reviews and reinforces an asthma action plan with a client who has moderate persistent asthma. Which statement by the client indicates an understanding of how to follow a plan appropriately when peak expiratory flow (PEF) readings are in the green, yellow, or red zones? a. If I am in the green zone (PEF 80%-100% of personal best) but I am coughing, wheezing, and having more trouble breathing, I will not make any changes in my medications b. If I am in the yellow zone (50%-80%) and I return to the green zone after taking my rescue medication, I will not make any changes in my daily medications c. If I am in the yellow zone (50%-80%), I will take my rescue medication every 4 hours for 1-2 days and call my health care provider for follow up care d. If I remain in the red zone, my lips are blue, and my PEF is still <50% of my personal best reading after taking my rescue medication, I will wait 15 minutes before calling the ambulance

An asthma action plan is an individualized management plan developed collaboratively between the client and the HCP to facilitate self-management of asthma. It includes information on daily and long-term treatment, prescribed medicines and when to take them according to a zone system, how to manage worsening symptoms or attacks, and when to call the HCP or go to the emergency department. The action plan uses traffic signal colors to categorize into zones degrees of asthma symptom severity and airway obstruction (peak flow meter readings): Green zone indicates asthma is under control and PEF is 80%-100% of personal best. When in this zone, there is no worsening of cough, wheezing, or trouble breathing (Option 1). Yellow zone means caution; even on a return to the green zone after use of rescue medication, further medication or a change in treatment is needed (Option 2). Red zone indicates a medical alert and signals the need for immediate medical treatment if the level does not return to yellow immediately after taking rescue medications (Option 4). Educational objective:A peak flow meter uses traffic signal colors to categorize degrees of asthma symptoms. Green zone indicates asthma is under control. Yellow zone indicates caution, symptoms are getting worse, PEF is 50%-80% of personal best, and there is a need for further medication. Red zone indicates the need for emergency treatment if the level does not immediately return to yellow after taking rescue medications.

The nurse in the outpatient procedure unit is caring for a client immediately post bronchoscopy. Which assessment data indicate that the nurse needs to contact the health care provider immediately? a. Absence of gag reflex b. Bright red blood mixed with sputum c. Headache d. Respirations 10/min and saturation of 92%

An endoscopic bronchoscopy is a procedure in which the bronchi are visualized with a flexible fiberoptic bronchoscope that is passed down through the nose (or through the mouth, or endotracheal or tracheostomy tube). The client receives mild sedation (eg, midazolam) to provide relaxation and promote comfort. A topical anesthetic (eg, lidocaine, benzocaine) is applied to the nares and throat to suppress the gag and cough reflexes, prevent laryngospasm, and facilitate passage of the scope. The procedure is done to diagnose, obtain tissue samples for biopsy, lavage, and to remove secretions (mucus plugs), foreign objects, or abnormal tissue with a laser. Blood-tinged sputum is common and can occur from inflammation of the airway, but hemoptysis of bright red blood can indicate hemorrhage, especially if a biopsy was performed. Other complications include hypoxemia, hypercarbia, hypotension, laryngospasm, bradycardia, pneumothorax (rare), and adverse effects from medications used before and during the procedure. (Option 1) Absence of the gag reflex for about 2 hours following the procedure is expected from the topical anesthetic. (Option 3) Headache is not a complication of bronchoscopy. (Option 4) Respirations of 10/min and saturation of 92% are expected after mild sedation before and/or or during the procedure. Educational objective:Immediately post bronchoscopy, the nurse monitors for associated potential complications, including hemoptysis, hypoxemia, hypercarbia, hypotension, laryngospasm, bradycardia, pneumothorax, and adverse effects from medications used before and during the procedure.

A nurse receives report on a group of clients. Which client should the nurse assess first? a. A preschool age child with a harsh cough, expiratory wheezes, and mild intercostal retractions b. A toddler playing with small toys who appears in distress, has circumoral cyanosis, and cannot speak c. A toddler with a barking cough, infrequent inspiratory stridor, and O2 saturation of 94% on RA d. An infant with an axillary temp of 100.1 F who is tugging at the left ear.

Aspiration of a foreign body occurs most often in the toddler age group. Swallowing of objects such as buttons, small parts of toys, or food particles can be life-threatening and result in airway obstruction due to the small diameter of the airway. Manifestations include choking, gagging, cyanosis, and inability to speak when the object is lodged in the larynx. (Option 1) Although the client has mild retractions with wheezing and a harsh cough, a patent airway is present. This client may be experiencing expected manifestations of asthma, but this is not a life-threatening condition. (Option 3) The client's manifestations are consistent with laryngotracheobronchitis (croup), which is generally caused by a parainfluenza virus. There is no respiratory challenge indicated by a 94% oxygen saturation on room air, and this not an emergency situation. (Option 4) Otitis media is an infection or inflammation of the middle ear with the highest incidence at age 6-36 months; it occurs during the winter months. Acute onset presents with ear pain, irritability, fever, and pulling on the affected ear. Fluid can accumulate in the middle ear and create an environment for bacterial growth. Respiratory distress is not seen. Educational objective:Using the priorities of airway, breathing, and circulation, maintenance of airway function requires immediate intervention by a nurse.

A mother reports to the pediatric nurse that her 3-year-old child coughs at night and at times until he vomits. The symptoms have not improved over the past 2 months despite multiple over-the-counter cough medications. What should the nurse explore related to a possible etiology? a. Ask about exposure to triggers such as pet dander b. Assess for the presence of butterfly rash c. Hx of intolerance to wheat food products d. Palpate for an abdominal mass from pyloric stenosis

Asthma is a chronic inflammatory disease of the lungs in genetically susceptible children. Frequent cough, especially at night, is the warning signal that the child's airway is very sensitive to stimuli; it may be the only sign in "silent" asthma. Common triggers include indoor contaminants (eg, tobacco smoke, pet dander, cockroach feces), outdoor contaminants (eg, air pollution), and allergic disease (eg, hay fever, food allergies). (Option 2) A red or pink butterfly rash across the cheeks and bridge of the nose is classic for systemic lupus erythematosus (SLE), an autoimmune disease that affects connective tissue. The child has no symptoms of SLE. Manifestations are acute (eg, nephritis, arthritis, vasculitis) or involve a gradual onset of nonspecific symptoms. (Option 3) Celiac, or gluten-sensitive, enteropathy is a chronic malabsorption syndrome. There is intolerance for gluten, a protein found in wheat, barley, rye, and oats. This condition affects absorption of nutrients; it does not cause nausea. (Option 4) Pyloric stenosis is a hypertrophy of the pylorus that results in stenosis of the passage between the stomach and the duodenum. Symptoms become evident 2-8 weeks after birth. It starts with occasional vomiting that eventually becomes forceful/projectile vomiting as the obstruction becomes complete. Dehydration and electrolyte imbalance result. The thickened pyloric muscle can sometimes be palpated and can be confirmed with ultrasound. This child is too old for this complication. Educational objective:Pediatric asthma can present as night coughing until the child vomits.

A pediatric client is diagnosed with an acute asthma attack. Which immediate-acting medications should the nurse prepare to administer to this client? Select all that apply. a. Albuterol b. Ibuprofen c. Ipatropium d. Montelukast e. Tobramycin

Asthma is an inflammatory condition in which the smaller airways constrict and become filled with mucus. Breathing, especially on expiration, becomes more difficult. Pharmacologic treatment for acute asthma includes the following: Oxygen to maintain saturation >90% High-dose inhaled short-acting beta agonist (albuterol or levalbuterol) and anticholinergic agent (ipratropium) nebulizer treatments every 20 minutes Systemic corticosteroids (Solu-Medrol) to control the underlying inflammation. These will take some time to show an effect. (Option 2) Nonsteroidal anti-inflammatory agents (eg, ibuprofen, naproxen, indomethacin) and aspirin can worsen asthma symptoms in some clients and are not indicated unless necessary. (Option 4) Montelukast (Singulair) is a leukotriene (chemical mediator of inflammation) inhibitor and is not used to treat acute episodes. It is given orally in combination with beta agonists and corticosteroid inhalers (eg, fluticasone, budesonide) to provide long-term asthma control. (Option 5) Tobramycin is an aminoglycoside antibiotic. It is used in aerosolized form to treat cystic fibrosis exacerbation when Pseudomonas is the predominant organism causing lung infection. Educational objective:Inhaled corticosteroids and leukotriene inhibitors are typically used to achieve and maintain control of inflammation for long-term management of asthma. Quick-relief medications (eg, albuterol, ipratropium) are used to treat acute symptoms and exacerbations.

A client is admitted with an exacerbation of asthma following a respiratory viral illness. Which clinical manifestations characteristic of a severe asthma attack does the nurse expect to assess? Select all that apply. a. Accessory muscle use b. Chest tightness c. High pitched expiratory wheeze d. Prolonged inspiratory phase e. Tachypnea

Asthma is an obstructive lung disease characterized by hyperreactive airways and chronic inflammation. Asthma exacerbations occur due to various triggers (eg, allergens, respiratory infection, exercise, cold air), resulting in edema, hypersecretion of mucus, and bronchospasm. Narrowing of the airways culminates in increased airway resistance, air trapping, and lung hyperinflation. In severe asthma, breath sounds may be diminished due to closure of bronchioles. Absent breath sounds in a client with asthma are a medical emergency. Clinical manifestations of an asthma exacerbation include: Accessory respiratory muscle use related to increased work of breathing and diaphragm fatigue (Option 1) Chest tightness related to air trapping (Option 2) Cough from airway inflammation and increased mucus production Diminished breath sounds related to hyperinflation High-pitched expiratory wheezing caused by narrowing airways (Option 3); wheezing may be heard on both inspiration and expiration as asthma worsens Tachypnea related to inability to take a full, deep breath (Option 5) (Option 4) Clients with obstructive lung disease (eg, asthma, chronic obstructive pulmonary disease) develop prolonged expiratory phase as a physiologic response to hyperinflation and trapped air. Educational objective:Asthma is an obstructive lung disease characterized by hyperreactive airways and chronic inflammation. Clinical manifestations of an asthma exacerbation include accessory respiratory muscle use, chest tightness, diminished breath sounds, high-pitched wheezing on expiration, prolonged expiratory phase, tachypnea, and cough.

The nurse in a clinic is caring for an 8-month-old with a new diagnosis of bronchiolitis due to respiratory syncytial virus (RSV). Which instructions can the nurse anticipate reviewing with the parent? a. Administering a cough suppressant and antihistamine b. Prophylactic treatment of family members c. Temporary cessation of breastfeeding d. Use of saline drops and a bulb syringe to suction nares

Bronchiolitis is a common viral illness of childhood that is usually caused by RSV. It typically begins with viral upper respiratory symptoms (eg, rhinorrhea, congestion) that progress to lower respiratory tract symptoms such as tachypnea, cough, and wheezing. Bronchiolitis is a self-limited illness and supportive care is the mainstay of treatment. Most children can be managed in the home environment. Breastfeeding should be continued and additional fluids offered if there is a risk of dehydration due to frequent coughing and vomiting (Option 3). Parents should be instructed to use saline nose drops and then suction the nares with a bulb syringe to remove secretions prior to feedings and at bedtime (Option 4). (Option 1) Medications such as cough suppressants, antihistamines, bronchodilators (eg, albuterol), and corticosteroids have not been found to be effective and are not recommended. (Option 2) Prophylactic treatment of family members is recommended for pertussis infection but not for RSV bronchiolitis. Educational objective:Bronchiolitis is a common viral illness of childhood that is usually caused by RSV. The focus of home care is on monitoring respiratory status and periodic nasal suctioning using saline nose drops to ease breathing. Additional fluids should be offered.

The nurse auscultates the lung sounds of a newly admitted client. The nurse understands that the lung sounds heard are consistent with which health condition? Listen to the audio clip. (Headphones are required for best audio quality.) a. Bronchitis b. Croup c. Pleurisy d. Pneumothorax

Bronchitis is inflammation of the upper airways (bronchi) often precipitated by a viral infection. Rhonchi (ie, sonorous wheeze) are continuous, low-pitched adventitious breath sounds that occur when thick secretions or foreign bodies (eg, tumors) obstruct airflow in the upper airways. The resulting sound resembles moaning or snoring and is heard primarily during expiration but may also be present during inspiration. Rhonchi are commonly heard in bronchitis, cystic fibrosis, or some types of pneumonia, and may clear with coughing or suctioning (Option 1). (Option 2) Croup often manifests with stridor, a high-pitched inspiratory breath sound that can often be heard without using a stethoscope. Stridor is caused by partial obstruction of the upper airway and is often louder over the throat. (Option 3) Pleurisy manifests with pleural friction rub, a loud, rough rubbing or grating sound heard throughout inspiration and expiration that is caused by the pleural surfaces rubbing together. Pleural friction rub sounds similar to crackles, but crackles are typically heard only during inspiration. (Option 4) Lung sounds are diminished or absent with pneumothorax due to compression of lung tissue by air in the pleural space. Educational objective:Rhonchi are continuous, low-pitched adventitious breath sounds similar to moaning or snoring that occur when thick secretions or foreign bodies (eg, tumors) obstruct airflow in the upper airways, as in bronchitis.

Which medication prescriptions should the nurse question? Select all that apply. a. Cephalexin for a client with severe allergy to penicillin b. Fexofenadine for a client with hives c. Ibuprofen for a client with asthma and nasal polyps d. Lisinopril for a client with diabetes mellitus e. Propranolol for a client with asthma

Cephalexin is a cephalosporin, which is chemically similar to penicillin. If a client has had a severe allergic reaction to penicillin, there is a 1%-4% chance of an allergic reaction (cross-sensitivity) to a cephalosporin (Option 1). Clients with nasal polyps often have sensitivity to nonsteroidal anti-inflammatory drugs (NSAIDS), including aspirin. In addition, NSAIDs can exacerbate asthma symptoms. Therefore, acetaminophen may be a better choice for these clients (Option 3). The selective beta blockers (eg, metoprolol, atenolol, bisoprolol) are generally given for heart failure and hypertension control due to their beta1-blocking effect. The nonselective beta blockers (eg, propranolol, nadolol), in addition, have a beta2-blocking effect that results in bronchial smooth muscle constriction. Therefore, nonselective beta blockers are generally contraindicated in clients with asthma (Option 5). (Option 2) H1 receptor antagonists (eg, fexofenadine, cetirizine, levocetirizine, loratadine) decrease the inflammatory response by blocking histamine receptors. Histamine is released from mast cells during a type I (immediate) hypersensitivity reaction (ie, allergic rhinitis, allergic conjunctivitis, and hives). (Option 4) Angiotensin-converting (ACE) inhibitors (ending in "pril") are the drugs of choice in diabetic clients with hypertension or proteinuria. This would be an appropriate administration. Educational objective:Clients with asthma and nasal polyps can have sensitivity to NSAIDs; those with an allergy to penicillin can have a cross-sensitivity to cephalosporins. Nonselective beta blockers are contraindicated in clients with asthma. H1 receptor antagonists block histamine in an allergic reaction. ACE inhibitors are protective for diabetic nephropathy.

The nurse is caring for a 4-year-old client with cystic fibrosis who uses a high-frequency chest wall oscillation (HFCWO) vest for chest physiotherapy. After reinforcing education with the client's parents, which statement by a parent requires further teaching? a. I will allow my child to have a snack while using the HFCWO vest to encourage cooperation b. I will give my child the nebulized bronchodilator tx during therapy with the HFCWO c. I will perform manual chest percussion on my child if the HFCWO is broken or unavailable d. My child will use the HFCWO vest once in the morning, once in the evening, and as needed

Chest physiotherapy (CPT) describes techniques of airway clearance, which is an important component of treatment for clients with cystic fibrosis that loosens and drains thick respiratory secretions. CPT can be performed by percussing (ie, clapping) the chest with a cupped hand or by wearing an inflatable high-frequency chest wall oscillation (HFCWO) vest. The HFCWO vest inflates and deflates rapidly, causing vibration over the chest wall and mobilizing secretions into the large airways that the child can expectorate. The HFCWO vest's rapid vibrations may induce nausea and vomiting in some clients. Therefore, the client should avoid meals and snacks 1 hour before, during, or 2 hours following CPT to prevent gastrointestinal upset (Option 1). The nurse may suggest other more appropriate ways to ensure compliance with CPT, such as allowing the child to watch a favorite television show or reading the child a story while wearing the HFCWO vest. (Option 2) Nebulized bronchodilators are often given before or during CPT treatments to open the airways and mobilize secretions. (Option 3) CPT can be administered using various methods, including percussion (ie, clapping) of the chest wall with cupped hands. (Option 4) CPT should be performed at least twice a day, and more often if needed. Educational objective:Chest physiotherapy (CPT) is an important component of treatment for clients with cystic fibrosis. A high-frequency chest wall oscillation vest is a common method of performing CPT, but treatments should occur 1 hour before or 2 hours after eating to avoid gastrointestinal upset (eg, nausea, vomiting).

A client with chronic bronchitis tells the home health nurse of being exhausted all day due to coughing all night and being unable to sleep. The client can feel thick mucus in the chest and throat. Which interventions can the nurse suggest to help mobilize secretions and improve sleep? Select all that apply. a. Increase fluids to at least 8 glasses (2-3 L) of water a day b. Sleep with a cool mist humidifier c. Take prescribed guaifenesin cough medicine before bedtime d. Use abdominal breathing and the huff cough technique at bedtime e. Use pursed lip breathing during the night

Chronic bronchitis is characterized by excessive mucus production, chronic cough, and recurrent respiratory tract infections. Interventions to help reduce viscosity of mucus, facilitate secretion removal, and promote comfort include the following: Increasing oral fluids to 2-3 L/day if not contraindicated prevents dehydration and keeps secretions thin Cool mist humidifier increases room humidity of inspired air Guaifenesin (Robitussin) is an expectorant that reduces the viscosity of thick secretions by increasing respiratory tract fluid; drinking a full glass of water after taking the medication is recommended. Abdominal breathing with the huff, a forced expiratory cough technique, is effective in mobilizing secretions into the large airways so that they can be expectorated Chest physiotherapy (postural drainage, percussion, vibration) Airway clearance handheld devices, which use the principle of positive expiratory pressure to help loosen secretions when the client exhales through the mouthpiece (Option 5) Pursed lip breathing prolongs exhalation, reduces air trapping in the lungs, and decreases dyspnea. It does not help to thin secretions. Educational objective:Interventions to help reduce viscosity of mucus, facilitate secretion removal, and promote comfort in clients with chronic bronchitis include the following: Increasing oral fluids to 2-3 L/day if not contraindicated Cool mist humidifier to increase room humidity Guaifenesin (Robitussin), an expectorant, to reduce viscosity of secretions Huff coughing

The nurse takes the admission history of a 70-year-old client diagnosed with chronic obstructive pulmonary disease (COPD). Which of the following statements by the client does the nurse recognize as contributing to the development of COPD? Select all that apply. a. I have been drinking alcohol almost daily since age 20 b. I have been overweight for as long as I can remember c. I have smoked about a pack of cigarettes a day since I was 16 years old but quit last year d. I know I eat too much fast food e. I was a car mechanic for about 40 years and had my own garage

Chronic obstructive pulmonary disease (COPD) generally refers to two conditions: emphysema and chronic bronchitis. A combination of the two is common. COPD affects approximately 12 million people and is the third leading cause of death in the United States, occurring most commonly in the seventh decade of life. It is characterized by slowly progressive, persistent airflow obstruction, and its etiology is closely associated with chronic airway inflammation. The major risk factor for COPD is current or former tobacco smoking (eg, cigarette, pipe, cigar). An additional contributing factor is prolonged exposure to respiratory irritants (eg, chemical fumes, smoke, dust) related to the client's occupation (eg, car mechanic, firefighter, coal miner). Risk for COPD is even higher if the client both smokes tobacco and has occupational exposure to respiratory irritants (Options 3 and 5). Chronic exposure to air pollution and genetic predisposition (eg, alpha1-antitrypsin deficiency) also contribute. (Option 1) Alcohol consumption is not associated with the development of COPD. (Options 2 and 4) Although obesity can worsen COPD symptoms by contributing to dyspnea, obesity and poor nutrition are not factors that directly contribute to the development of COPD. Educational objective:Chronic airway inflammation is closely associated with the development of chronic obstructive pulmonary disease. Specific etiologic factors include current or former tobacco smoking, prolonged exposure to occupational respiratory irritants, chronic exposure to air pollution, and genetic predisposition.

A home health nurse visits a client with chronic obstructive pulmonary disease. The nurse teaches the client to use abdominal breathing to perform the "huff" cough technique to facilitate secretion removal. Place the steps in the correct order. All options must be used. a. Hold your breath for 2-3 seconds and then forcefully exhale quickly b. Sit upright in a chair with feet spread shoulder-width apart and lead forward c. Repeat the huff once or twice more, while refraining from performing a normal cough d. Perform a slow, deep, inhalation with your mouth using your diaphragm. e. Rest for 5-10 normal breaths and repeat as necessary until mucus is cleared.

Clients with chronic obstructive pulmonary disease (COPD) often develop ineffective coughing patterns due to weakened muscles and narrowed airways prone to collapse under increased pressure. Therefore, clients with COPD are unable to cough effectively and require additional teaching to effectively expectorate secretions and prevent overexertion. Huff coughing is a series of low-pressure coughs using the following steps: Sit upright in a chair with feet spread shoulder-width apart and lean forward with shoulders relaxed; forearms supported on thighs or pillows; head and knees slightly flexed; and feet touching the floor (Option 5). Perform a slow, deep inhalation through the mouth or nose using the diaphragmatic muscle (Option 2). Hold breath for 2-3 seconds, keeping the throat open, and then perform a quick, forceful exhalation, creating an audible "huff" sound (Option 1). Repeat the "huff" once or twice more to expectorate any mucus (Option 3). Rest for 5-10 regular breaths and repeat as necessary until all mucus is cleared (Option 4). Educational objective:Clients with chronic obstructive pulmonary disease benefit from breathing techniques to facilitate effective coughing. Huff coughing is a forced expiratory technique in which the client sits relaxed, upright, and leaning forward; slowly inhales using the diaphragmatic muscle; holds breath for 2-3 seconds and then quickly exhales; and repeats as necessary until remaining secretions are clear.

The nurse reviews discharge instructions with a client who has advanced chronic obstructive pulmonary disease. Which client statement indicates appropriate understanding? Select all that apply. a. I need to take iron supplements to prevent anemia b. I should report an increase in sputum c. I will eat a low-calorie diet d. I will get a pneumococcal vaccine e. I will use albuterol if I am short of breath

Clients with chronic obstructive pulmonary disease (COPD) suffer from progressive inflammatory tissue damage, which eventually leads to lung scarring and airway remodeling. Through these mechanisms, COPD leads to chronic air trapping and reduced gas exchange by decreasing ventilation. Clients with COPD are at increased risk for respiratory infections, which can trigger an acute exacerbation of COPD. Therefore, it is vital that clients receive both routine influenza and pneumococcal vaccinations (Option 4). It is also important that clients seek medical help for increased sputum (Option 2), worsening shortness of breath, or lack of relief from prescribed emergency medications (eg, albuterol, ipratropium) (Option 5). (Option 1) COPD may lead to polycythemia (increased red blood cells), in which the body attempts to compensate for chronic hypoxia by increased proliferation of erythrocytes. This occurs when erythropoietin is released from the kidneys in response to hypoxemia and leads to erythropoiesis. This ultimately has the opposite effect of anemia, making supplementation with iron not necessary and possibly even harmful. (Option 3) Clients with COPD have increased work of breathing and are often winded by simple activities (eg, eating). Clients should eat frequent, small, high-calorie meals to conserve energy and meet nutritional requirements. Educational objective: Clients with chronic obstructive pulmonary disease should be instructed to consume a high-calorie diet, seek medical attention for signs of infection (eg, increased sputum, worsening dyspnea, fever), and obtain appropriate vaccinations (eg, influenza, pneumococcal) to prevent exacerbations.

A client is having a severe asthma attack lasting over 4 hours after exposure to animal dander. On arrival, the pulse is 128/min, respirations are 36/min, pulse oximetry is 86% on room air, and the client is using accessory muscles to breathe. Lung sounds are diminished and high-pitched wheezes are present on expiration. Based on this assessment, the nurse anticipates the administration of which of the following medications? Select all that apply. a. Inhaled albuterol nebulizer every 20 minutes b. Inhaled ipratopium nebulizer every 20 minutes c. Methyprednisolone IV d. Montelukast 10 mg PO STAT e. Salmetrol MDI every 20 minutes

Clinical manifestations characteristic of moderate to severe asthma exacerbations include tachycardia (>120/min), tachypnea (>30/min), saturation <90% on room air, use of accessory muscles to breathe, and peak expiratory flow (PEF) <40% of predicted or best (<150 L/min). Pharmacologic treatment modalities recommended by the Global initiative for Asthma (2014) to correct hypoxemia, improve ventilation, and promote bronchodilation include the following: Oxygen to maintain saturation >90% High-dose inhaled short-acting beta agonist (SABA) (albuterol) and anticholinergic agent (ipratropium) nebulizer treatments every 20 minutes Systemic corticosteroids (Solu-Medrol) (Option 4) Montelukast (Singulair) is a leukotriene receptor blocker with both bronchodilator and anti-inflammatory effects; it is used to prevent asthma attacks but is not recommended as an emergency rescue drug in asthma. (Option 5) A long-acting beta agonist (Salmeterol) is administered with an inhaled corticosteroid for long-term control of moderate to severe asthma; it is not used as an emergency rescue drug in asthma. Educational objective:Clinical manifestations characteristic of moderate to severe asthma exacerbations include tachycardia, tachypnea, saturation <90% on room air, use of accessory muscles of respiration, and PEF <40% predicted. Management includes the administration of high-dose inhaled SABA and ipratropium nebulizer, systemic corticosteroids, and oxygen to maintain saturation >90%.

The nurse auscultates the lung sounds of a client with shortness of breath. Based on the sounds heard, which action would the nurse anticipate? Listen to the audio clip. (Headphones are required for best audio quality.) Review sound of Coarse crackles on you tube a. Administer albuterol via nebulizer b. Administer furosemide IV push c. Instruct to use pursed lip breathing d. Prepare for chest tube insertion

Coarse crackles (loud, low-pitched bubbling) are heard primarily during inspiration and are not cleared by coughing. The sound is similar to that of Velcro being pulled apart. Coarse crackles may be confused with fine crackles (eg, atelectasis), which have a high-pitched popping sound. Coarse crackles are present when fluid or mucus collects in the lower respiratory tract (eg, pulmonary edema, pulmonary fibrosis). In heart failure, the left ventricle fails to eject enough blood, causing increased pressure in the pulmonary vasculature. As a result, fluid leaks into the alveoli (pulmonary edema). Diuretics (eg, furosemide) treat pulmonary edema by increasing fluid excretion by the kidneys (Option 2). (Option 1) Clients with asthma or chronic obstructive pulmonary disease (eg, emphysema) develop wheezing due to bronchospasm. Bronchodilators (eg, albuterol, ipratropium) are indicated for these clients. (Option 3) Emphysema is a chronic hyperinflation of the alveoli. Clients with emphysema are taught the pursed-lip breathing technique to prevent alveolar collapse during exhalation. Emphysema causes diminished lung sounds, prolonged expiration, and wheezing. (Option 4) Chest tubes are inserted into the pleural space to remove trapped air (eg, pneumothorax) or fluids (eg, hemothorax, pleural effusion). Lung sounds are diminished or absent when lung tissue is compressed by air or fluids in the pleural space. Educational objective:Auscultation of coarse crackles indicates the presence of fluid or mucus in the lower respiratory tract. This may indicate pulmonary edema or pulmonary fibrosis. Diuretic administration (eg, furosemide) is used to treat pulmonary edema.

A nurse has received new medication prescriptions for a client admitted with hypertension and an exacerbation of chronic obstructive pulmonary disease. Which prescription should the nurse question? a. Amlodipine b. Codeine c. Ipratropium d. Methyprenisolone

Codeine is a narcotic analgesic used for acute pain or as a cough suppressant. Depressing the cough reflex can cause an accumulation of secretions in the presence of chronic obstructive pulmonary disease (COPD), leading to respiratory difficulty. In general, sedatives (eg, narcotics, benzodiazepines) can also depress the respiratory center and effort; therefore, they should not be given to clients with respiratory diseases (eg, asthma, COPD). (Option 1) Calcium channel blockers (eg, amlodipine, nifedipine) are used to treat hypertension and do not worsen bronchoconstriction, unlike beta blockers (eg, metoprolol, atenolol). (Option 3) Ipratropium (Atrovent) is a short-acting inhaled anticholinergic often used in combination with a short-acting beta-agonist (eg, albuterol) to promote bronchodilation and reduce bronchospasm. (Option 4) Methylprednisolone (Solu-Medrol) is a systemic glucocorticoid that improves respiratory symptoms and overall lung function in clients experiencing an exacerbation of COPD. Educational objective:Codeine is a narcotic medication with antitussive properties that can cause an accumulation of secretions in clients with chronic obstructive pulmonary disease and lead to respiratory distress. Caution is advised when sedatives are prescribed for clients with respiratory diseases.

An elderly client is prescribed codeine for a severe cough. The home health nurse teaches the client how to prevent the common adverse effects associated with codeine. Which client statements indicate an understanding of how to prevent them? Select all that apply. a. I'll be sure to apply sunscreen if I go outside b. I'll drink at least 8 glasses of water a day c. I'll drink decaffeinated coffee so I can sleep at night d. I'll sit on the side of my bed for a few minutes before getting up e. I'll take my medicine with food.

Codeine is an opioid drug prescribed as an analgesic to treat mild to moderate pain and as an antitussive to suppress the cough reflex. Although the antitussive dose (10-20 mg orally every 4-6 hours) is lower than the analgesic dose, clients can still experience the common adverse effects (eg, constipation, nausea, vomiting, orthostatic hypotension, dizziness) associated with the drug. Codeine decreases gastric motility, resulting in constipation. Increasing fluid intake and fiber in the diet and taking laxatives are effective measures to prevent constipation (Option 2). Changing position slowly is effective in preventing the orthostatic hypotension associated with codeine, especially in the elderly (Option 4). Taking the medication with food is effective in preventing the gastrointestinal irritation (eg, nausea, vomiting) associated with codeine (Option 5). (Options 1 and 3) These statements are inaccurate as photosensitivity, insomnia, palpitations, and anxiety are not adverse effects associated with codeine. Educational objective:The common adverse effects of codeine, an opioid drug, include constipation, nausea, vomiting, orthostatic hypotension, and dizziness. Interventions to help prevent them include increasing fluid intake and bulk in the diet, laxatives, taking the medication with food, and changing position slowly.

A client with chronic obstructive pulmonary disease reports recent weight loss and poor appetite. The client states that bloating, exhaustion, and shortness of breath make eating "not worth the effort." Which statements by the nurse are appropriate to help improve the client's nutritional status? Select all that apply. a. Avoid drinking fluids while you are eating meals b. Eat small, frequent meals that are high in calories and protein c. Exercise before you eat, to improve you appetite d. Increase your intake of high fiber foods such as broccoli and cabbage e. Perform oral hygiene before eating meals

Consuming adequate nutrition is difficult for clients with advanced chronic obstructive pulmonary disease (COPD), as chewing and swallowing increase work of breathing and a full stomach increases pressure on the diaphragm. As a result, clients often lose weight because their energy expenditure is greater than their nutritional intake. To optimize nutritional intake, clients should: Drink fluids between meals, rather than before or during, to prevent stomach distension and decrease pressure on the diaphragm while eating (Option 1). Eat small, frequent meals, snacks, and supplements that are high in calories and protein. Smaller meals require less energy to chew and swallow, resulting in less fatigue and dyspnea (Option 2). Perform oral hygiene before meals. Chronic mouth breathing leads to dry mouth; excessive sputum and medication side effects can alter the taste of food, decreasing the appetite (Option 5). (Option 3) For clients with advanced COPD, exercise is discouraged for 1 hour before and 1 hour after eating as it increases oxygen demand and fatigue. (Option 4) Gas-forming foods (eg, broccoli, beans, cabbage) and carbonated beverages should be avoided as they cause intestinal bloating and increased pressure on the diaphragm. Educational objective:Consuming adequate nutrition is difficult for clients with advanced chronic obstructive pulmonary disease as eating increases dyspnea and fatigue. To optimize nutrition, clients should avoid drinking fluids while eating; eat small, frequent, high-calorie, high-protein meals and snacks; avoid exercising for 1 hour before and 1 hour after eating; and perform oral hygiene before meals.

The nurse cares for a child newly diagnosed with cystic fibrosis. What should be included in the client's multidisciplinary plan of care to be discussed with the parents? Select all that apply. a. Aerobic exercise b. Chest physiotherapy c. Financial needs d. Low-calorie diet e. Oral fluid restriction

Cystic fibrosis (CF) is a genetic disorder involving the cells lining the respiratory, gastrointestinal (GI), and reproductive tracts. A defective protein responsible for transporting sodium and chloride causes secretions in these areas to be thicker and stickier than normal. These abnormal secretions plug smaller airway passages and ducts in the GI tract, which can impair digestive enzymes and result in ineffective absorption of essential nutrients. These sticky respiratory secretions lead to a chronic cough and inability to clear the airway, eventually causing chronic lung disease (bronchiectasis). As a result of these changes, the client's life span is shortened; most affected individuals live only into their 30s. Chest physiotherapy helps remove sticky secretions that cause ineffective airway clearance (Option 2). Aerobic exercise is beneficial to promote removal of airway secretions, improve muscle strength, and increase lung capacity (Option 1). Financial needs must be discussed, as clients with CF have a large financial burden due to health care costs, medications, and special equipment (Option 3). (Option 4) A diet high in fat and calories is recommended due to defective digestive enzymes and impaired nutrient absorption. (Option 5) Fluids are not restricted; liberal intake is recommended to assist in thinning respiratory secretions. Educational objective:Clients with cystic fibrosis should have a diet high in fat and calories to combat nutrient malabsorption. Liberal fluid intake is encouraged to loosen thick secretions. Chest physiotherapy and aerobic exercise are performed to remove airway secretions. Financial needs are addressed as clients have a large financial burden.

An obese 85-year-old client, who is an avid gardener and eats only home-grown fruits, legumes, and vegetables, is admitted to the hospital with pneumonia after having an upper respiratory tract infection for a week. Which factor puts the client at greatest risk for developing pneumonia? a. Advanced age b. Environmental exposure c. Nutritional deficit d. Obesity

Pneumonia is an inflammatory process in the alveoli and interstitium of the lung usually caused by an infectious or noninfectious agent. Any condition, such as advanced age (>65), that compromises the respiratory system's protective mechanical or immune mechanisms to maintain the sterility of the lower airway can increase the risk for pneumonia. (Options 2, 3, and 4) Working in the garden and being exposed to environmental factors (eg, pollen), eating a vegetarian diet, and obesity do not pose the greatest risks for development of pneumonia. Educational objective:Any condition that compromises the respiratory system's protective mechanical or immune mechanisms, which maintain the sterility of the lower airway, can increase the risk for pneumonia.

The nurse assesses a client with a history of cystic fibrosis who is being admitted due to a pulmonary exacerbation. Which assessment finding requires immediate action by the nurse? a. Decrease in SpO2 from baseline 92% to 88% on room air b. Expectorating blood tinged sputum c. Loss of appetite and recent 5 lb weight loss d. No bowel movement for 2 days and right lower quadrant discomfort

Cystic fibrosis (CF) is a genetic disorder in which a defective protein causes the exocrine glands to produce thick and sticky secretions that can plug small airways and ducts (eg, gastrointestinal tract). Respiratory manifestations include ineffective airway clearance, risk of infection, and eventually chronic lung disease (bronchiectasis). CF also damages alveoli, which can burst suddenly, resulting in pneumothorax and need for chest tube placement. Clients with CF often have low baseline oxygen saturation; however, a sudden drop in SpO2 (eg, from 92% to 88% on room air) indicates a possible mucus plug airway obstruction and the need for urgent intervention (eg, physiotherapy, mucolytic medications) (Option 1). (Option 2) Pulmonary capillaries can be damaged during CF exacerbations, resulting in hemoptysis (bloody sputum). Gross hemoptysis (eg, frank blood) can be life threatening; however, blood-tinged sputum is not a priority finding. (Option 3) Impaired secretion of digestive enzymes results in malabsorption of essential nutrients. In addition, many clients have a poor appetite. Maintaining weight is important; however, oxygenation is the priority. (Option 4) Although impaired digestion and absorption cause frequent steatorrhea (fatty stool) episodes, clients with CF can develop distal intestinal obstruction syndrome (DIOS) from dehydrated, thickened mucus and stool. DIOS causes constipation and abdominal discomfort that can be resolved by rehydrating the stool (eg, polyethylene glycol). Educational objective:Clients with cystic fibrosis are at risk for respiratory infection, mucus plugs, and pneumothorax. When respiratory status suddenly declines (eg, drop in SpO2), urgent intervention (eg, physiotherapy, chest tube) is required to resolve mucus plug airway obstruction.

A home health nurse is managing care for an adolescent client with cystic fibrosis. Which of the following potential complications should the nurse consider when developing a nursing care plan? Select all that apply. a. Chronic hypoxemia b. Diabetes Insipidus c. Frequent respiratory infections d. Obesity e. Vitamin deficiencies

Cystic fibrosis (CF) is an inherited disorder (autosomal recessive) characterized by thickened secretions due to impaired chloride and sodium channel regulation that causes exocrine gland dysfunction. Management of a client with CF should primarily address potential complications related to the following body systems: Pulmonary: Alterations in respiratory secretions (ie, thick sputum) make it difficult to clear the airway and can result in frequent respiratory infections and sinusitis (Option 3). Frequent infections and inflammation damage lung tissue and may lead to chronic hypoxemia (Option 1). Gastrointestinal: Thickened secretions obstruct the release of pancreatic enzymes, causing malabsorption of fat-soluble vitamins (eg, A, E, D, K) and other nutritional deficiencies (Option 5). High-protein, high-calorie foods and supplemental enzymes with meals are necessary. Reproductive: Thickened reproductive secretions (eg, seminal fluid, cervical mucus) or the absence of the vas deferens in men contributes to CF-related infertility. (Option 2) Diabetes mellitus, not diabetes insipidus, is a potential complication for clients with CF due to pathologic pancreatic changes (eg, fibrosis). (Option 4) Due to impaired gastrointestinal absorption, weight loss and failure to thrive are more common and a greater concern than obesity. Educational objective:Cystic fibrosis is an inherited disorder that results in impaired exocrine gland function and is characterized by thickened secretions that affect the pulmonary, gastrointestinal, and reproductive systems. When planning care, the nurse should monitor for priority concerns, including development of respiratory infections, chronic hypoxemia, nutritional deficiencies, and abnormal growth (failure to thrive).

A 6-month-old client has been diagnosed with cystic fibrosis. Which of the following would be appropriate for the registered nurse to teach to the parents? a. Monitor for and report development of a "white pupil" b. Perform manual chest physiotherapy c. Place child in knee-chest position during hypercyanotic episode d. Provide low calorie diet to prevent obesity

Cystic fibrosis is an inherited autosomal recessive disorder of the exocrine glands that results in physiologic alterations in the respiratory, gastrointestinal, and reproductive systems. It is theorized that the chloride transport alternation and resulting thickened mucus inhibit normal ciliary action and cough clearance, and the lungs become clogged with mucus. The thickened mucus harbors bacteria. Over time, airways develop chronic colonization and frequent respiratory infections result. Bronchial hygiene therapy, such as manual chest physiotherapy, is used. For physiotherapy, various positions are used, and this should be performed before meals to avoid a full stomach and resultant regurgitation or vomiting. (Option 1) A white pupil (leukocoria, or cat's-eye reflex) is one of the first signs of retinoblastoma, an intraocular malignancy of the retina. Other symptoms include an absent red reflex, asymmetric or of a differing color in the affected eye, and fixed strabismus (constant deviation of one eye from the other). This disease is not related to cystic fibrosis. (Option 3) Hypercyanotic episodes are associated with tetralogy of Fallot. The knee-chest position increases systemic vascular resistance in the lower extremities. In addition, irritating stimuli should be limited, and supplemental oxygen should be provided. (Option 4) The pancreatic ducts become damaged, and there is a decreased ability to digest fats and proteins and absorb fat-soluble vitamins. Pancreatic enzyme supplements are used. Children with cystic fibrosis tend to be hungry but underweight due to a decreased ability to use fat and its calories. Educational objective:Cystic fibrosis causes thickened mucus, making respiratory infections common. Treatment includes chest physiotherapy performed usually before meals.

The nurse is caring for a 72-year-old client 1 day postoperative colectomy. The nurse assesses an increased work of breathing, diminished breath sounds at the bases with fine inspiratory crackles, respirations 12/min and shallow, and pulse oximetry 96% on 2 L oxygen. There is no jugular venous distension or peripheral edema. Pain is regulated with client-controlled morphine. Which prescription does the nurse anticipate? a. Bolus dose of IV morphine b. Incentive spirometer c. IV furosemide d. Non rebreather mask

During the initial postoperative period, a client needs respiratory interventions to keep the lungs expanded and prevent atelectasis and postoperative pneumonia. Atelectasis is maximal during the second postoperative night. Clients can be asymptomatic or have increased work of breathing, hypoxia, and basal crackles. Postoperative pain, opioid respiratory depression, limited mobility, and reluctance to take a deep breath due to anticipated pain contribute to postoperative atelectasis. The elderly and postoperative abdominal and thoracic surgery clients are at increased risk for atelectasis. The incentive spirometer encourages the client to breathe deeply with maximum inspiration. This action improves ventilation and oxygenation by expanding the lungs, encourages coughing, and prevents or improves atelectasis. It is the most appropriate prescription for this client. (Option 1) In a client whose pain is regulated with client-controlled analgesia (eg, morphine), administration of a bolus dose is not indicated and may increase the risk for respiratory depression. (Option 3) Fine crackles in the lungs usually indicate atelectasis. The presence of coarse crackles, elevated jugular venous distension, and peripheral edema usually indicates volume overload (fluid in the alveoli). In addition, clients with fluid overload breathe at a rapid rate (tachypnea) rather than take slow, shallow breaths. IV furosemide (Lasix) is an appropriate intervention for volume overload but not for atelectasis. (Option 4) As-needed oxygen may be prescribed postoperatively, especially with blood loss. A non-rebreather mask, which has 100% oxygen, is not indicated in this client as the pulse oximeter shows 96% saturation, indicating adequate oxygenation. Educational objective:The incentive spirometer is a handheld, inexpensive breathing device. It encourages the client to breathe deeply with maximum inspiration, which improves ventilation and oxygenation and encourages coughing. The incentive spirometer is used to prevent or improve atelectasis in clients who are postoperative, have respiratory problems (eg, pneumonia), or have experienced trauma.

A client has an allergy skin test that is positive for dust mites. The nurse provides instruction on environmental interventions the client can use to control symptoms by reducing exposure to this allergen. Which intervention would be described in this teaching? a. Allergy shots or sublingual immunotherapy b. Antihistamine use c. Vacuum carpeting once a week d. Wash bed linens in hot water once a week

Environmental interventions can be very effective in controlling dust mite allergy symptoms. The bed is a common site of allergen exposure. Dust mite allergen exposure can be greatly reduced by washing bed linens every 1-2 weeks with hot water. High temperature (>140 F [60 C]) is needed to kill the dust mites; warm or cold water washing should not be recommended. Other environmental interventions that can help control symptoms include the use of special allergy-proof mattress and pillow covers and vacuuming the mattress on a regular basis. (Option 1) Allergy shots and sublingual immunotherapy can reduce allergy symptoms by exposing the immune system to the allergen over time, reducing the immune response with subsequent exposure. However, this is not an environmental intervention. (Option 2) Antihistamines can be effective in controlling allergy symptoms. However, they are not an environmental intervention. (Option 3) If possible, clients with allergy or asthma should avoid having carpeting in the home. If carpeting is used, vacuuming should be done almost daily (not weekly) to remove the dust mites. Educational objective:Environmental interventions for reducing exposure to dust mite allergen in the bed include frequently washing linens with hot (>140 F [60 C]) water and using special mattress and pillow covers.

The nurse is assessing a 3-year-old client in the emergency department and finds dyspnea, high fever, irritability, and open-mouthed drooling with leaning forward. The parents report that the symptoms started rather abruptly. The client has not received age-appropriate vaccinations. Which set of actions should the nurse anticipate? a. 20 gauge needle insertion at the mid axillary line for pleural aspiration b. 4 L oxygen at 100% n/c with BPAP ventilation standing by c. Intubation in the operating room with prepared tracheotomy kit standing by d. Nebulized racemic epinephrine with pediatric anesthesiologist standing by

Epiglottitis should be considered first in a 3-7-year-old child with acute respiratory distress, toxic appearance (eg, sitting up, leaning forward, drooling), stridor, and high-grade fever. Tachycardia and tachypnea are also present. This is a pediatric emergency and should be managed with endotracheal intubation; however, intubation of such clients is difficult, and preparation for possible tracheostomy is also standard. The complications of epiglottitis are serious and include sudden airway obstruction. (Option 1) This is a recommended therapy for spontaneous tension pneumothorax, which is demonstrated by tracheal deviation; absent lung sounds; and severe, abrupt hypotension and dyspnea. (Option 2) Neither oxygenation nor BPAP is acceptable in acute epiglottitis as the trachea can close completely from edema. (Option 4) This is the appropriate therapy for croup, not epiglottitis. Croup is notably distinct for the hacking cough, which does not occur in epiglottitis. Educational objective:When assessing a client with symptoms suggestive of epiglottitis (eg, acutely ill, drooling, leaning forward, dyspnea), the nurse should prepare for an emergency airway.

The nurse receives change of shift report on 4 clients. Which client should the nurse assess first? a. 6 month old with RSV and pulse ox of 90% b. 1 year old with otitis media and a temp of 102.5 c. 2 year old with suspected epiglottitis d. 3 year old who has a barking type cough

Epiglottitis, a sudden-onset medical emergency due to Haemophilus influenzae, causes severe inflammatory obstruction above and around the glottis. The affected child will typically progress from having no symptoms to having a completely occluded airway within hours. Sitting in a tripod position (upright and leaning forward with the chin and tongue sticking out) is a classic presentation. The child will likely drool and be very restless and anxious secondary to airway obstruction and hypoxia. Throat inspection should not be done until emergency intubation is readily available (if necessary). (Option 1) Oxygen saturation ≥90% is the treatment goal for bronchiolitis caused by respiratory syncytial virus. (Option 2) This temperature is an expected finding in the setting of otitis media and does not carry the urgency of airway impairment. (Option 4) A barking-type cough is seen in viral croup syndromes. The resonant hoarse cough is secondary to narrowed airways. Croup is typically mild but can become life-threatening if the airway swells excessively. This child would need to be assessed next. Educational objective:Epiglottitis is a medical emergency as the child can rapidly progress from being asymptomatic to having a completely occluded airway. Emergency intubation equipment should be readily available.

A client with type 2 diabetes, coronary artery disease, and peripheral arterial disease developed hospital-acquired pneumonia (HAP) and has been receiving intravenous (IV) antibiotics for 4 days. Which parameter monitored by the nurse best indicates the effectiveness of treatment? a. Color of sputum b. Lung sounds c. Saturation level d. WBC

HAP is a bacterial infection acquired in a health care facility that was not present on admission. Almost all clients with bacterial pneumonia develop leukocytosis (WBC >11,000/mm3). Antibiotic therapy is the first-line treatment, but antibiotic resistance frequently occurs in HAP. If antibiotic therapy is effective, clinical improvement usually occurs within 3-4 days of initiation of IV antibiotics. The nurse monitors WBC as the best indicator of treatment effectiveness as antibiotics cause bacterial lysis or hinder bacterial DNA reproduction. The reduced number of bacteria and the resulting decrease in inflammation cause a decrease in the number of white blood cells needed to fight the infection. Other indicators of treatment effectiveness include improvement of infiltrates on chest x-ray, oxygenation, and signs and symptoms (cough, fever, sputum production). (Option 1) The color of sputum (eg, clear, yellow, green, grey, rusty, blood-tinged) can vary with different types of pneumonia; it is not the best indicator of treatment effectiveness. (Option 2) Adventitious/abnormal lung sounds (crackles, low-pitched wheeze, bronchial breath sounds) can be present as the pneumonia resolves or can be a sign of further complication (pleural effusion). However, these are not the best indicators of treatment effectiveness. (Option 3) Saturation is an indicator of oxygenation but can be affected by many other factors, such as coexisting disease, peripheral circulation, and drugs. It is not the best indicator of treatment effectiveness. Educational objective:Indicators of treatment effectiveness for HAP include decreased WBC on complete blood count with differential and improvement of infiltrates on chest-x-ray, oxygenation, and signs and symptoms (cough, fever, sputum production).

A client had a thoracotomy 2 days ago to remove a lung mass and has a right chest tube attached to negative suction. Immediately after turning the client to the left side to assess the lungs, the nurse observes a rush of approximately 125 mL of dark bloody drainage into the drainage tubing and collection chamber. What is the appropriate nursing action? Click on the exhibit button for additional information. Laboratory and output values Preoperative - Hemoglobin 15 g/dL Postoperative Day 1 - Hgb -12.5 g/dL; Chest drainage output - 400 ml/24 hr Post operative Day 2 Hgb - 13 g/dL; Chest drainage output - 50 ml/12 hr a. Document and continue to monitor chest drainage b. Immediately clamp the chest tube c. Notify the HCP d. Request repeat Hct and Hgb levels

Immediately following a thoracotomy, chest tube drainage (50-500 mL for the first 24 hours) is expected to be sanguineous (bright red) for several hours and then change to serosanguineous (pink) followed by serous (yellow) over a period of a few days. A rush of dark bloody drainage from the chest tube when the client was turned following a period of minimal drainage is most likely related to retained blood due to a partial blockage in the tube. Bright red drainage indicates active bleeding and would be of immediate concern. (Option 2) The chest tube should not be clamped because it is placed to drain the fluid leaking after surgery. (Option 3) The nurse would notify the health care provider immediately of bright red drainage or continued increased drainage (>100 mL/hr) and of changes in the client's vital signs and cardiovascular status that could indicate bleeding (eg, hypotension, tachycardia, tachypnea, decreased capillary refill, cool and pale skin). This is not the appropriate action. (Option 4) It would be appropriate to request repeat serum hematocrit and hemoglobin levels if active bleeding is suspected, but the postoperative levels are stable at this time. This is not the appropriate action. Educational objective:A client will usually have a chest tube in place for several days following a thoracotomy to drain blood from the pleural space. A rush of dark bloody drainage from the tube when the client coughs, turns, or is repositioned following a period of minimal drainage is most likely related to retained blood due to a partial blockage in the tube. Bright red chest drainage indicates active bleeding and would be of immediate concern.

The nurse assesses these symptoms in a client with bacterial pneumonia: chills, elevated temperature, tachypnea, productive cough of yellow sputum, shortness of breath, and fatigue. Based on the assessment data, what is the most appropriate nursing diagnosis (ND) for this client? a. Impaired Gas Exchange b. Impaired spontaneous ventilation c. Ineffective breathing pattern d. Risk for infection

Impaired gas exchange is a deficit in oxygenation and/or elimination of carbon dioxide at the alveolar-capillary membrane. Impaired gas exchange related to a bacterial infectious process as evidenced by shortness of breath and tachypnea is an appropriate ND for a client with pneumococcal pneumonia. (Option 2) Impaired spontaneous ventilation is the inability to maintain independent ventilation to support life and requires mechanical ventilation. Based on this client's assessment data, it is not an appropriate ND. (Option 3) This client is demonstrating an ineffective breathing pattern; however, this problem is secondary to impaired gas exchange. An increased respiratory rate is the body's attempt to compensate for hypoxia caused by consolidations and secretions preventing adequate gas exchange in the lungs. Impaired gas exchange is the primary problem that is causing the ineffective respirations and is the more appropriate ND for this client. (Option 4) Risk for infection is the increased risk for invasion of microorganisms. However, this client has an actual, not potential infection, so this is not an appropriate ND. Educational objective:Impaired gas exchange related to a bacterial infectious process as evidenced by shortness of breath and tachypnea is an appropriate nursing diagnosis for a client with pneumonia.

A nurse is caring for a college athlete who was recently diagnosed with moderate persistent asthma. Which common asthma trigger should the nurse teach this client to avoid? a. Latex containing products b. Penicillin antibiotics c. Second hand cigarette smoke d. Strenoud physical activity

In clients with asthma, the airways are chronically inflamed with varying degrees of airway obstruction that can be exacerbated by exposure to triggering agents. Common asthma triggers include: Allergens: Dander (eg, cat, dog), dust mites, pollen Drugs: Beta blockers; nonsteroidal anti-inflammatory agents, including aspirin Environmental: Chemicals, sawdust, soaps/detergents Infectious: Upper respiratory infections Intrinsic: Emotional stress, gastrointestinal reflux disease Irritants: Aerosols/perfumes, cigarette smoke (including secondhand smoke), dry/polluted air (Option 3) Clients must be able to identify their individual triggers and avoid or learn to manage them. (Options 1 and 2) Penicillin antibiotics and latex-containing products may commonly trigger allergic reactions in many clients but do not commonly trigger asthma exacerbations in clients without these allergies. (Option 4) Although physical activity is an asthma trigger, athletes with asthma do not need to avoid activity altogether. Rather, they may take an inhaled bronchodilator 20 minutes before activity to help prevent exercise-induced asthma attacks. In addition, this client may be prone to minor musculoskeletal injuries (eg, sprains, strains) due to an active lifestyle; the nurse should teach about alternatives to common over-the-counter nonsteroidal anti-inflammatory medications that may be used for analgesia (eg, acetaminophen [Tylenol]). Educational objective:The nurse should teach an active young adult with asthma to identify and manage common triggers of asthma attacks, including cigarette smoke and nonsteroidal anti-inflammatory medications. Clients with asthma should take an inhaled bronchodilator 20 minutes before athletic activity to prevent exercise-induced asthma attacks.

The nurse has provided teaching about home care to the parent of a 10-year-old with cystic fibrosis. Which of the following statements by the parent indicates that teaching has been effective? Select all that apply. a. Chest physiotherapy is administered only if respiratory symptoms worsen b. I will give my child pancreatic enzymes with all meals and snacks c. I will increase my child's salt intake during hot weather d. Our child will need a high-carbohydrate, high protein diet e. We will limit out child's participation in sports activities

In clients with cystic fibrosis (CF), a defective protein responsible for transporting sodium and chloride causes exocrine gland secretions to be thicker and stickier than normal. Viscous respiratory secretions accumulate, resulting in impaired airway clearance and a chronic cough. Clients eventually develop chronic lung disease, which predisposes them to recurrent respiratory infections. Pancreatic enzyme secretion, needed for digestion and absorption of nutrients, is also impaired because thick secretions block pancreatic ducts. Therefore, the client needs supplemental enzymes with all meals and snacks (Option 2). The client also requires multiple vitamins and a diet high in carbohydrates, protein, and fat to help meet nutritional requirements for growth (Option 4). Sweat gland abnormalities prevent sodium and chloride reabsorption, causing increased salt loss, dehydration, and hyponatremia during times of significant perspiration. Therefore, parents should increase the child's salt intake and fluids during hot weather, exercise, or fever (Option 3). (Option 1) Regardless of symptoms, clients should incorporate chest physiotherapy (eg, percussion, vibration, postural drainage) into their daily routine to improve mucus clearance and lung function. (Option 5) The parents should encourage physical activity as tolerated, which helps to thin secretions and remove them from airways and improves muscle strength and lung capacity. Educational objective:Cystic fibrosis causes increased viscosity of exocrine gland secretions. Clients require pancreatic enzyme supplements with meals and snacks; a diet high in carbohydrates, protein, and fat; and increased salt intake during times of significant perspiration. Clients should also incorporate chest physiotherapy and exercise into their daily routine.

The nurse is reviewing discharge instructions with the parents of a child who just had a tracheostomy. Which statement made by the parents indicates teaching has been effective? a. I will always travel with two tracheostomy tubes, one of the same size and one a size smaller b. I will immediately change the tracheostomy tube if my child has difficulty breathing c. I will provide deep suctioning frequently to prevent any airway obstruction d. I will remove the humidifier if my child starts developing more secretions

In the event of an accidental decannulation or another urgent need to change a tracheostomy tube, the most important action is to quickly replace the tube as it is the client's only means to ventilate. Clients should always carry two spare tracheostomy tubes, one the same size and one a size smaller. If the tube is not easily replaced or is meeting resistance, the smaller tube should be used. (Option 2) Changing a tracheostomy tube is a high-risk procedure that should be done only if respiratory distress is noted and other interventions (eg, suctioning) have failed. Mucus plugs (ie, thickening and buildup of mucus due to dehydration) are one of the most common causes of respiratory distress. (Option 3) A tracheostomy should be suctioned frequently to maintain airway patency. However, deep suctioning should be reserved for clients in respiratory distress due to the risk of injury. Tracheostomy tubes should be suctioned to the specified depth using a measurement marked on the tube, to provide safe, effective suctioning. (Option 4) Humidification is crucial for clients with a tracheostomy as the upper airway, which provides natural humidity for inhaled air, is bypassed. Humidification helps keep secretions thin and reduces formation of mucus plugs. The humidifier should not be removed if the child develops more secretions as this is the intended effect. Educational objective:Clients with a tracheostomy should always carry two spare tubes, one the same size and one a size smaller, to ensure that the tube can be replaced quickly and effectively.

The nurse in the postanesthesia care unit (PACU) is caring for an unresponsive client who just came from the operating room following surgery under general anesthetic for colorectal cancer. The nurse chooses what as the highest priority nursing diagnosis (ND)? a. Acute pain b. Impaired physical mobility c. Ineffective airway clearance d. Risk for fluid volume deficit.

Ineffective airway clearance, which is the inability to clear secretions or obstructions from the respiratory tract to maintain a clear airway, is the priority ND as it poses the greatest threat to survival. The most common causes of respiratory complications in the immediate postoperative period include the following: Airway obstruction, which can be due to retained secretions or the tongue falling backward against the soft palate in sedated clients. Suctioning and an artificial oral airway can be used to prevent obstruction until the client becomes more responsive. Hypoxemia, which can be due to atelectasis from increased retained secretions or hypoventilation, aspiration, or bronchospasm. Pulse oximetry and supplemental oxygen are used to maintain pulse oximeter readings >92%; placing the client in side-lying position and administrating antiemetic medications help to decrease aspiration. Hypoventilation, which can be due to depression of the respiratory drive as a result of anesthesia, pain, and opioid analgesia. (Option 1) Most clients receive continual or bolus epidural or patient-controlled opioid analgesia to control postoperative pain. Pain assessment after surgery is a priority, and acute pain is an appropriate ND. However, it does not pose the greatest threat to survival and is not the priority diagnosis at this time. (Option 2) Ambulation and ability to reposition are important interventions to prevent multiple postoperative complications. However, impaired physical mobility is not the priority ND at this time. (Option 4) Risk for fluid volume deficit related to fluid losses during surgery is an appropriate diagnosis, but it does not pose the greatest threat to survival and is not the priority ND at this time. Educational objective:Acute pain, impaired physical mobility, ineffective airway clearance, and risk for fluid volume deficit are appropriate NDs in a client in the PACU who is immediately postoperative. The highest priority ND is the one that poses the greatest threat to survival based on the client's current health status.

The client has a chest tube for a pneumothorax. While repositioning the client for an x-ray, the technician steps on the tubing and accidently pulls the chest tube out. The client's oxygen saturation drops and the pulse is 132/min; the nurse hears air leaking from the insertion site. What is the nurse's immediate action? a. Apply an occlusive sterile dressing secured on 3 sides b. Apply an occlusive sterile dressing secured on 4 sides c. Assess lung sounds d. Notify the HCP

a. Apply an occlusive sterile dressing secured on 3 sides If the chest tube is dislodged from the client and the nurse hears air leaking from the site, the nurse's immediate action should be to apply a sterile occlusive dressing (eg, petroleum jelly dressing) taped on 3 sides. This action permits air to escape on exhalation and inhibits air intake on inspiration. The nurse would then notify the HCP and arrange for the reinsertion of another chest tube (Option 1). (Option 2) A tension pneumothorax develops when air enters the pleural space but cannot escape. Increased intrapleural pressure and excessive accumulation of air can apply pressure to the heart and great vessels and drastically decrease cardiac output. An occlusive dressing taped on 4 sides would prevent the air in the pleural space from escaping on exhalation and would increase the risk for a tension pneumothorax. (Option 3) The nurse would stay with the client, assess lung sounds, and monitor vital signs frequently; however, this is not the immediate action. (Option 4) The nurse would notify the HCP and prepare for reinsertion of a chest tube, but it is not the immediate action. Educational objective:If a chest tube is dislodged from the client and the nurse hears air leaking from the site, the immediate action should be to apply an occlusive sterile dressing taped on 3 sides. This action decreases the risk for a tension pneumothorax by inhibiting air intake on inspiration and allowing air to escape on expiration.

The community health nurse prepares a teaching plan for a client with latent tuberculosis who is prescribed oral isoniazid (INH). Which instructions should the nurse include? Select all that apply. a. Avoid drinking alcohol b. Expect body fluids to change color to red c. Report yellowing of skin or sclera d. Report numbness and tingling of the extremeties e. Take with aluminum hydroxide to prevent gastric irritation

Isoniazid (INH) is a first-line antitubercular drug prescribed as monotherapy to treat latent tuberculosis infection. Combined with other drugs, INH is also used for active tuberculosis treatment. Two serious adverse effects of INH use are hepatotoxicity and peripheral neuropathy. A teaching plan for a client prescribed INH includes the following: Avoid intake of alcohol and limit use of other hepatotoxic agents (eg, acetaminophen) to reduce risk of hepatotoxicity (Option 1) Take pyridoxine (vitamin B6) if prescribed to prevent neuropathy Avoid aluminum-containing antacids (eg, aluminum hydroxide (Maalox)) within 1 hour of taking INH Report changes in vision (eg, blurred vision, vision loss) Report signs/symptoms of severe adverse effects such as:Hepatoxicity (eg, scleral and skin jaundice, vomiting, dark urine, fatigue) (Option 3)Peripheral neuropathy (eg, numbness, tingling of extremities) (Options 4) (Option 2) Rifampin, another antitubercular drug, often causes a red-orange discoloration of body fluids (ie, urine, sweat, saliva, tears). However, this effect is not associated with INH use. (Option 5) Concurrent use of antacids containing aluminum decreases INH absorption. The medication may be taken with food if gastric irritation is a concern. Educational objective:Common potential side effects of INH include hepatotoxicity (eg, jaundice, vomiting, dark urine, fatigue) and peripheral neuropathy (eg, numbness, tingling of extremities). Clients should avoid alcohol use and aluminum-containing antacids, and report any experienced side effects to the health care provider immediately.

A nurse is reviewing the laboratory results of a client admitted for an asthma exacerbation. Elevation of which of these cells indicates that the client's asthma may have been triggered by an allergic response? a. Eosinophils b. Lymphocytes c. Neutrophils d. Reticulocytes

Normal eosinophil count is 1%-2%. Elevated eosinophils are seen in allergy. In a client with an asthma exacerbation, a high eosinophil count would indicate an allergic trigger for the asthmatic response. The nurse should explore the client's allergy history and ways to reduce the allergic exposure that may be contributing to the exacerbation. (Option 2) Lymphocytes form the major part of immune system. Elevated levels are seen with viral infections and hematologic malignancies. (Option 3) Normal neutrophils are 55%-70%. Elevated neutrophils indicate infection. (Option 4) Reticulocytes are immature red blood cells. Normal reticulocyte count is 0.5%-2.0%. Levels are elevated in hemolytic anemia or hemorrhage when the marrow is attempting to compensate for lost blood. Educational objective:An elevated eosinophil count in the complete blood count is associated with allergy. Allergies are frequently triggers of asthma exacerbation.

A hospitalized client with a history of obstructive sleep apnea sleeps while wearing a full face mask with continuous positive airway pressure (CPAP). Oxygen saturation drops to 85% during the night. What is the nurse's first action? a. Assess level of consciousness and lung sounds b. Check the tightness of the straps and mask c. Notify the HCP d. Remove the mask and administer supplemental oxygen

Obstructive sleep apnea (OSA) is a chronic condition that involves the relaxation of pharyngeal muscles during sleep. The resulting upper airway obstruction with multiple events of apnea and shallow breathing (hypopnea) leads to hypoxemia and hypercapnia. CPAP is an effective treatment for OSA; it involves using a nasal or full face mask that delivers positive pressure to the upper airway to keep it open during sleep. In this case, the nurse's first action should be to check the tightness of the straps that hold the mask in place. The full face mask must fit snugly over the client's nose and mouth without air leakage to maintain the positive airway pressure and prevent obstruction of upper airway airflow. Readjustment of the head straps may be necessary (Option 2). (Option 1) Underlying OSA is the most likely reason for this client's drop in oxygen saturation during sleep. If CPAP is not effective, then the characteristic OSA signs (eg, hypoxia, hypercapnia) will occur. In addition, decreased level of consciousness and lung sounds are expected when there is no airflow to the lungs. Although the nurse should assess these parameters, this should not be the first action. (Option 3) If the attempt to readjust the straps and mask seal does not reverse the client's hypoxemia quickly, the nurse should notify the health care provider and respiratory therapist (per institution policy). However, this should not be the nurse's first action. (Option 4) Supplemental oxygen may be indicated if readjustment of the straps and mask seal does not reverse the client's hypoxemia quickly. This should not be the nurse's first action. Educational objective:CPAP is prescribed for clients with obstructive sleep apnea in the home and clinical settings. The mask is secured with adjustable head straps to maintain a snug fit over the face to prevent air leakage and loss of positive pressure.

The nurse is teaching an overweight 54-year-old client about ways to decrease symptoms of obstructive sleep apnea. Which interventions would be most effective? Select all that apply. a. Eating a high-protein snack at bedtime b. Limiting alcohol intake c. Losing weight d. Taking a mild sedative at bedtime e. Taking a modafinil at bedtime f. Taking a nap during the day

Obstructive sleep apnea (OSA) is characterized by partial or complete airway obstruction during sleep that occurs from relaxation of the pharyngeal muscles. The result is repeated episodes of apnea (≥10 seconds) and hypopnea (≤50% normal ventilation), which cause hypoxemia and hypercarbia. Common symptoms include frequent periods of sleep disturbance, snoring, morning headache, daytime sleepiness, difficulty concentrating, forgetfulness, mood changes, and depression. Interventions include: Continuous positive airway pressuredevice at night to keep the structures of the pharynx and tongue from collapsing backward Limiting alcohol intake at bedtime as it can cause muscles of the oral airway to relax and lead to airway obstruction (Option 2) Weight loss and exercise can reduce snoring and sleep apnea-associated airway obstruction. Obesity contributes to the development of OSA (Option 3). Avoiding sedating medications (eg, benzodiazepines, sedating antidepressants, antihistamines, opiates) as they may exacerbate OSA and worsen daytime sleepiness (Option 1) Eating before bedtime can interfere with sleep and contribute to excess weight. (Option 4) Sedatives at bedtime can relax the muscles of the oral airway and lead to airway obstruction. (Option 5) Stimulants such as modafinil may be prescribed for daytime sleepiness but should be avoided at bedtime as they can cause insomnia. (Option 6) Napping during the day can make it more difficult to sleep through the night. Educational objective:Obstructive sleep apnea is characterized by partial or complete airway obstruction during sleep. Interventions to relieve symptoms include a continuous positive airway pressure device during sleep and lifestyle changes (eg, weight loss; exercise; avoiding food, alcohol, and sedatives at bedtime).

The nurse is gathering data on a client with obstructive sleep apnea. Which findings are consistent with this client's diagnosis? Select all that apply. a. Difficulty arousing from sleep b. Excessive daytime sleepiness c. Morning headaches d. Postural collapse and falling e. Snoring during sleep f. Witness episodes of apnea

Obstructive sleep apnea (OSA) is the most common type of breathing disorder during sleep and is characterized by repeated periods of apnea (>10 seconds) and diminished airflow (hypopnea). A partial or complete obstruction occurs due to upper airway narrowing that results from relaxation of the pharyngeal muscles or from the tongue falling back on the posterior pharynx due to gravity. During periods of apnea, desaturation (hypoxemia) and hypercapnia occur; these stimulate the client to arouse and breathe momentarily to restore airflow. These cycles of apnea and restored airflow can occur several hundred times per night, resulting in restless and fragmented sleep. Partners of clients with OSA witness loud snoring, apnea episodes, and waking with gasping or a choking sensation (Options 5 and 6). During the day, clients experience morning headaches, irritability, and excessive sleepiness. Excessive daytime sleepiness can lead to poor work performance, motor vehicle crashes, and increased mortality (Options 2 and 3). (Option 1) Frequent (not difficult) arousal from sleep is associated with OSA. (Option 4) Cataplexy is a brief loss of skeletal muscle tone or weakness that can result in a client falling down. It is associated with narcolepsy, a chronic neurologic sleep disorder. Educational objective:At night, clients with obstructive sleep apnea experience repeated periods of apnea, loud snoring, and interrupted sleep. During the day, morning headaches, irritability, and excessive sleepiness are common.

The home health nurses visits a 72-year-old client with pneumonia who was discharged from the hospital 3 days ago. The client has less of a productive cough at night but now reports sharp chest pain with inspiration. Which finding is most important for the nurse to report to the health care provider? a. Bronchial breath sounds b. Increased tactile fremitus c. Low-pitched wheezing (rhonchi) d. Pleural friction rub

Pleurisy is characterized by stabbing chest pain that usually increases on inspiration or with cough. It is caused by inflammation of the visceral pleura (over the lung) and the parietal pleura (over the chest cavity). The pleural space (between the 2 layers) normally contains about 10 mL of fluid to help the layers glide easily with respiration. When inflamed, they rub together, causing pleuritic pain. A pleural friction rub is auscultated in the lateral lung fields over the area of inflammation. The sound is produced by the 2 layers rubbing together and can indicate pleurisy, a complication of pneumonia. It is characterized by squeaking, crackling, or the sound heard when the palm is placed over the ear and the back of the hand is rubbed with the fingers. Complications of pneumonia are more prevalent in elderly clients with underlying chronic disease. (Option 1) Clients with consolidative lung processes (pneumonia) may also have bronchial breath sounds due to over-transmission of sound over the chest wall. Breath sounds are diminished or absent over a pleural effusion or pneumothorax. (Option 2) Palpable vibration felt on the chest wall is known as fremitus. Sound travels faster in solids (consolidation) than in an aerated lung, resulting in increased fremitus in pneumonia. It is an expected finding in clients with pneumonia. It is concerning if decreased because the client may be developing pleural effusion. (Option 3) Low-pitched wheezing (rhonchi) is a continuous adventitious breath sound heard over the large airways, usually during expiration. It indicates the presence of secretions in the larger airways and is an expected finding as pneumonia resolves. Educational objective:Pleurisy is characterized by stabbing pleuritic chest pain that increases on inspiration. It is a complication of pneumonia caused by inflamed parietal and visceral pleurae rubbing together.

The nurse caring for a client with left lobar pneumonia responds to an alarm from the continuous pulse oximeter. The client is short of breath with an oxygen saturation of 78%. After applying oxygen, the nurse should place the client in which position to improve oxygenation? a. left lateral b. right lateral c. supine d. trendelenburg

Pneumonia is a lung infection resulting in decreased gas exchange in the affected lung lobes. The alveoli in the affected lobes become blocked with purulent fluid, which impairs ventilation. However, these alveoli continue to receive perfusion from the pulmonary artery, resulting in poorly oxygenated or deoxygenated blood. This ventilation-to-perfusion (V/Q) mismatch, or pulmonary shunt, may result in hypoxia and respiratory distress. Blood flow in the lungs is partially influenced by gravity, meaning that blood flows in higher volumes to dependent parts of the lung. Therefore, a client with left lobar pneumonia should be positioned in right lateral position with the unaffected (good) lung down (eg, right lung) to increase blood flow to the lung most capable of oxygenating blood (Option 2). (Option 1) Left lateral positioning will worsen hypoxia by decreasing blood flow to the unaffected (ie, right) lung. (Options 3 and 4) Positioning in supine or Trendelenburg position does not promote increased perfusion to the unaffected lung, which is needed to improve hypoxia. Educational objective:Pneumonia (ie, infection of the lungs) causes decreased gas exchange in the affected lung lobes, which can lead to hypoxia and respiratory distress. Clients with unilateral pneumonia should be positioned with the unaffected (ie, good) lung down to improve perfusion and oxygenation.

The nurse assesses a client with fever and productive cough for the last 10 days. Which findings support the presence of pneumonia? Select all that apply. a. Coarse crackles b. Hyperresonance c. Pleuritic chest pain d. Shortness of breath e. Trachea deviating from midline

Pneumonia is an acute infection of the lungs. Findings in a client with pneumonia include: Crackles - Fine or coarse crackling sounds caused by air passing through alveoli and small airways obstructed with mucus (Option 1) Fever, chills, productive cough, dyspnea, and pleuritic chest pain (Options 3 and 4) Increased vocal/tactile fremitus - Transmission of palpable vibrations (fremitus) is increased when transmitted through consolidated versus normal lung tissue. Bronchial breath sounds in peripheral lung fields - High-pitched, harsh sounds conducted through consolidated lung tissue, which are abnormal when heard in an area distant from where normally heard (ie, trachea); this finding can be an early sign of pneumonia. Unequal chest expansion - Decreased expansion of affected lung on palpation Dullness - Percussion of medium-pitched sounds over consolidated lung tissue (pneumonia) or fluid-filled space (eg, pleural effusion, a complication of pneumonia) (Option 2) Hyperresonance is percussed over a hyperinflated lung (eg, asthma, emphysema) or air in the pleural space (eg, pneumothorax). (Option 5) A trachea deviating from midline is not a symptom of pneumonia but instead indicates a tension pneumothorax where the trachea deviates away from the tension. Educational objective:Physical examination of a client with pneumonia can reveal crackles, increased vocal/tactile fremitus, unequal chest expansion, and bronchial breath sounds in peripheral areas. Clients often report fever, chills, productive cough, dyspnea, and pleuritic chest pain.

The nurse assesses a client with left-sided pneumonia who has an intermittent, productive cough with copious amounts of thick, yellow sputum. Which of the following interventions help to facilitate secretion removal? Select all that apply. a. Chest physiotherapy b. Cough suppressant c. Huff coughing technique d. Pursed lip breathing e. Right side-lying

Pneumonia is an inflammatory reaction in the lungs, often due to infection, that causes alveoli to fill with cellular debris and thick, purulent exudate (ie, consolidation), which may cause impaired ventilation and oxygenation. Interventions to facilitate secretion removal in clients with pneumonia include: Performing chest physiotherapy (percussion, vibration, postural drainage) to loosen and break up thickened secretions (Option 1) Assisting the client to perform huff coughing, which raises secretions from the lower to the upper airway for expectoration (Option 3) Ensuring adequate hydration through increased oral fluid intake (≥2-3 L/day) and administration of prescribed IV fluids, which thins pulmonary secretions to promote improved secretion clearance Positioning the head of the bed to 45-60 degrees (ie, Fowler position) to promote effective coughing and optimal lung expansion (Option 2) Cough suppressants reduce the urge to cough triggered by airway irritants (eg, purulent secretions) and ultimately impair secretion removal. (Option 4) Pursed-lip breathing prolongs exhalation and prevents airway collapse, which alleviates dyspnea relating to air trapping (eg, chronic obstructive pulmonary disease). However, it does not facilitate secretion removal. (Option 5) Side-lying positioning is utilized in hypoxic clients with unilateral pneumonia to increase perfusion to the healthy lung by gravity and improve oxygenation by positioning the client with the unaffected (good) side down. However, side-lying position alone does not improve secretion clearance Educational objective:Clients with pneumonia often develop thick secretions that impair oxygenation and require intervention to promote secretion clearance. Interventions to mobilize secretions include coughing and deep breathing, chest physiotherapy, Fowler positioning, and hydration.

A client with pneumonia is transferred from the medical unit to the intensive care unit due to sepsis and worsening respiratory failure. Based on the nurse's progress note, which assessment data are most important for the nurse to report to the health care provider (HCP)? Click on the exhibit button for additional information. Progress notes 1845 Productive cough of large amount of tan mucus, scattered rhonchi, and crackles in bases. Arterial blood gas (ABG) results: PaCO2 35 mm Hg, PaO2 90 mm Hg on nasal oxygen at 6L/min. Temperature 101.1 F (38.3 C). On vancomycin for 2 days.______________RN 1945 Repeat ABG: PaCO2 33 mm Hg, PaO2 89 mm Hg on 50% oxygen via face mask.______________RN 2045 Repeat ABG: PaCO2 32 mm Hg, PaO2 86 mm Hg on 100% oxygen via total rebreather mask.______________RN a. Cough with mucus production b. Refractory hypoxemia c. Scattered rhonchi and crackles d. Temp 101 F

Refractory hypoxemia is the hallmark of acute respiratory distress syndrome (ARDS), a progressive form of acute respiratory failure that has a high mortality rate. It can develop following a pulmonary insult (eg, aspiration, pneumonia, toxic inhalation) or nonpulmonary insult (eg, sepsis, multiple blood transfusions, trauma) to the lung. The insult triggers a massive inflammatory response that causes the lung tissue to release inflammatory mediators (leukotrienes, proteases) that cause damage to the alveolar-capillary (A-C) membrane. As a result of the damage, the A-C membrane becomes more permeable, and intravascular fluid then leaks into the alveolar space, resulting in a noncardiogenic pulmonary edema. The lungs become stiff and noncompliant, which makes ventilation and oxygenation less than optimal and results in increased work of breathing, tachypnea and alkalosis, atelectasis, and refractory hypoxemia. Profound hypoxemia despite high concentrations of oxygen is a key sign of ARDS and is the most important assessment finding to report to the HCP. (Options 1 and 3) Cough with mucus production and scattered rhonchi and crackles are expected findings in a client with pneumonia. (Option 4) Temperature is an expected finding in a client with pneumonia who is receiving antibiotic therapy. The white blood cell count can still be elevated after 2 days of antibiotic therapy. Educational objective:Refractory hypoxemia is the inability to improve oxygenation with increases in oxygen concentration. It is the hallmark of ARDS, a progressive form of acute respiratory failure that has a high mortality rate.

A client is brought to the emergency department due to loss of consciousness after binge drinking at a college party and then taking alprazolam. Pulse oximetry shows 87% on room air. Which findings would the nurse expect to assess on an arterial blood gas? a. Metabolic acidosis and hyperventilation b. Metabolic alkalosis and hypoventilation c. Respiratory acidosis and hypoventilation d. Respiratory alkalosis and hyperventilation

Respiratory acidosis and hypoventilation The combination of excessive alcohol ingestion and the benzodiazepine alprazolam (Xanax) causes respiratory depression, which leads to alveolar hypoventilation secondary to carbon dioxide retention, and respiratory acidosis. Therefore, clients should be advised not to take multiple substances that increase the risk of respiratory depression (eg, opioids, benzodiazepines, alcohol, sedating antihistamines). (Option 1) Diarrhea, ketoacidosis, lactic acidosis, and renal failure can cause metabolic acidosis due to loss of bicarbonate or retention of acids; the lungs would compensate by hyperventilating. (Option 2) Vomiting, gastrointestinal suction, and administration of alkali (ie, sodium bicarbonate) are common causes of metabolic alkalosis; the lungs would compensate by hypoventilating. (Option 4) Hypoxia, anxiety, and pain are common causes of respiratory alkalosis, which is due to alveolar hyperventilation (rapid breathing). Educational objective:Over-sedation, sleep apnea, anesthesia, drug overdose, progressive neuromuscular disease, and chronic obstructive pulmonary disease depress the respiratory center; this leads to alveolar hypoventilation, secondary to carbon dioxide retention, and respiratory acidosis.

The nurse is assessing a client diagnosed with tuberculosis who started taking rifapentine a week ago. Which statement by the client warrants further assessment and intervention by the nurse? a. I do not want to get pregnant, so I restarted my oral contraceptive last month. b. I have been taking my medications with breakfast every morning c. I should alert my HCP if I notice yellowing of my skin d. Since I started this medicine, my saliva has become a red-orange color.

Rifapentine (Priftin), a derivative of rifampin, is an antitubercular agent used with other drugs (eg, isoniazid) as a combination therapy in active and latent tuberculosis infections. Both rifampin and rifapentine reduce the efficacy of oral contraceptives by increasing their metabolism; therefore, this client will need an alternate birth control plan (non-hormonal) to prevent pregnancy during treatment (Option 1). (Option 2) Rifapentine should be taken with meals for best absorption and to prevent stomach upset. (Option 3) Hepatotoxicity may occur; therefore, liver function tests are required at least every month. Signs and symptoms of hepatitis include jaundice of the eyes and skin, fatigue, weakness, nausea, and anorexia. (Option 4) Rifapentine may cause red-orange-colored body secretions, which is an expected finding. Dentures and contact lenses may be permanently stained. Educational objective:Clients taking rifampin or rifapentine (Priftin) as part of antitubercular combination therapy should be taught to prevent pregnancy with non-hormonal contraceptives, notify the health care provider of any signs or symptoms of hepatotoxicity (eg, jaundice, fatigue, weakness, nausea, anorexia), and expect red-orange-colored body secretions.

The charge nurse evaluates the care provided by a new registered nurse (RN) for a client receiving mechanical ventilation (MV). Which action by the new RN indicates the need for further education? a. Administers morphine to relieve anxiety and restlessness b. Applies suction when inserting the catheter into the airway c. Increases the oxygen concentration on the MV before suctioning d. Suctions when MV high pressure alarm continues to sound and rhonchi are present

Risks associated with suctioning include hypoxemia, microatelectasis, and cardiac dysrhythmias. Suctioning removes secretions and oxygen. To minimize both the amount of oxygen removed and mucosal trauma, suction is applied when removing, not inserting, the catheter into the artificial airway. If secretions are thick and difficult to remove, increasing hydration, not suctioning time, is indicated. Aerosols of sterile normal saline or mucolytics such as acetylcysteine (Mucomyst) administered by nebulizer can also be used to thin the thick secretions, but water should not be used. Aerosol therapy may induce bronchospasm in certain individuals and can be relieved by use of a bronchodilator (albuterol). (Option 1) Morphine is administered to promote breathing synchrony with the mechanical ventilator, reduce anxiety, and promote comfort in clients receiving MV. (Option 3) Preoxygenation with 100% oxygen for 30 seconds before suctioning, unless otherwise specified, is the recommended practice to reduce suctioning-associated risks for hypoxemia, microatelectasis, and cardiac dysrhythmias. (Option 4) It is appropriate to suction the client when the high-pressure alarm on the MV sounds, saturations drop, rhonchi are auscultated, and secretions are audible or visible. These manifestations can indicate excessive secretions impairing airway patency. Educational objective:To minimize removal of oxygen and mucosal trauma, suction should be applied only when removing the catheter, not when inserting it. Other interventions to reduce the risks associated with suctioning (eg, hypoxemia, microatelectasis, cardiac dysrhythmias) include assessment for the need to suction, preoxygenating with 100% oxygen, and limiting suction time to 10-15 seconds.

The nurse provides instruction to a community group about lung cancer prevention, health promotion, and smoking cessation. Which statement made by a member of the group indicates the need for further instruction? a. Even though I am getting nicotine in my patches, I am bot being exposed to all the other toxic stuff in cigarettes b. I can't get lung cancer because I don't smoke c. My husband needs to take smoking cessation classess d. We installed a radon detector in our home

Smoking is responsible for 80%-90% of all lung cancers. Although the risk is greater among smokers, former smokers and nonsmokers can develop lung cancer as well. Risk factors include secondhand smoke, air pollution, genetic predisposition, and exposure to radon, asbestos, and chemicals in the workplace. (Option 1) Smoking cessation is the best way to prevent lung cancer. Nicotine replacement therapy (eg, patches, gum, inhalers, lozenges) is effective in helping smokers quit by reducing cravings. Although users receive a low dose of nicotine, they do not receive the other toxins that cigarettes include. (Option 3) The best way to reduce the risk of lung cancer is to avoid both firsthand and secondhand smoke. Smoke from someone else's burning cigarette contains the same carcinogens as those found in mainstream smoke and creates a health risk to those inhaling it. (Option 4) Exposure to high levels of radon can cause lung cancer. Radon levels must be tested before a home can be sold. Educational objective:Although the risk is greater among smokers, former smokers and nonsmokers can develop lung cancer. Risk factors include secondhand smoke, air pollution, genetic predisposition, and exposure to radon, asbestos, and chemicals in the workplace.

A client is transferred from the post-anesthesia recovery unit to the surgical unit following an open cholecystectomy. Which interventions are most important for the nurse to perform to prevent postoperative pneumonia? Select all that apply. a. Administer morphine only if the pain is >8/10 b. Ambulate within 8 hours after surgery, if possible c. Have client cough with splinting every hour d. Have client deep breathe and use the IS every hour e. Maintain pneumatic compression devices when client is in bed f. Place client in Fowler's position

Strategies to prevent postoperative pneumonia include the following: Adequate pain control is a priority so that the client can move, deep breathe, and cough more effectively and comfortably. Opioids are effective for relieving postoperative pain, but because they depress respirations and the cough reflex, assessing the client's response to the medication and level of sedation is important. Ambulate within 8 hours after surgery, if possible. Mobilization/early ambulation decreases atelectasis and hypoventilation, and promotes coughing, deep breathing, and lung expansion. Usually, it can be initiated within 4-8 hours after surgery. Coughing with splinting every hour. Splinting of the incision and adequate pain management are useful for promoting an effective cough (huff, cascade) that clears the airway of secretions. Deep breathing and use of the incentive spirometer every hour. Deep breathing in conjunction with the use of the incentive spirometer promotes ventilation and oxygenation. It opens the pores of Kohn that permit air from well-ventilated alveoli to move into collapsed alveoli, and it helps to prevent/decrease atelectasis and hypoventilation caused by the effects of anesthesia, analgesia, and pain. Place in Fowler's position. Elevating the head of the bed 45-60 degrees helps to promote oxygenation and prevent aspiration. Turn and reposition the client at least every 2 hours. Swab mouth with chlorhexidine swabs every 12 hours. Mouth care prevents ventilator-associated and postoperative pneumonia. Use hand hygiene (all personnel) to decrease transmission of microorganisms. (Option 1) Adequate pain control is a priority. The decision to hold opioids is usually based on sedation level. Giving pain medicine only for severe pain is not appropriate. If the client is awake and complains of pain, adequate analgesia should be provided (oral or IV). (Option 5) Use of pneumatic compression devices promotes venous return and helps to prevent venous thrombosis, not pneumonia. Educational objective:Strategies to prevent postoperative pneumonia include adequate pain management, positioning, coughing and deep breathing, mobilization and early ambulation, twice daily chlorhexidine mouth care, and precautions for limiting the transmission of microorganisms.

The school nurse assesses an 8-year-old with a history of asthma. The nurse notes mild wheezing and coughing. Which action should the nurse perform first? a. Assess the client's peak expiratory flow b. Call the HCP c. Educate the client about avoiding triggers d. Notify the client's parents

Symptoms of an asthma exacerbation include wheezing, chest tightness, dyspnea, cough (may be nocturnal, dry, or productive), and retractions. A cough is often the earliest sign of an asthma exacerbation in children. Bronchospasm leads to CO2 trapping and retention. The bronchospasm forces the client to work harder to exhale and the expiratory phase becomes prolonged. The nurse needs to further assess this client to validate the severity of the exacerbation before implementing an intervention. By assessing the client's peak expiratory flow, the nurse can determine the severity of the symptoms. The nurse will also need to assess the client's respiratory rate and lung sounds. (Option 2) Additional information is needed before notifying the HCP to determine the severity of the client's current condition. (Options 3 and 4) The client's parents do need to be notified and discuss asthma triggers with the nurse. However, these are not a priority as the client is currently symptomatic. Educational objective:The nurse must determine the severity of a client's condition before implementing an intervention. By assessing this client's peak expiratory flow, the nurse can determine the severity of the asthma symptoms.

The hospice nurse is caring for an actively dying client who is unresponsive and has developed a loud rattling sound with breathing ("death rattle") that distresses family members. Which prescription would be most appropriate to treat this symptom? a. Atropine sublingual drops b. Lorazepam sublingual tablet c. Morphine sublingual liquid d. Ondansetron sublingual tablet

The "death rattle" is a loud rattling sound with breathing that occurs in a client who is actively dying. When the client cannot manage airway secretions, the movement of these secretions during breathing causes a noisy rattling sound. This can distress family and friends at the bedside of the dying client. The "death rattle" can be treated using anticholinergic medications to dry the client's secretions. Medications include atropine drops administered sublingually or a transdermal scopolamine patch. (Option 2) Lorazepam is a benzodiazepine that is used to treat anxiety and restlessness in terminally ill clients. It can be effective for alleviating dyspnea exacerbated by anxiety, but it is ineffective for controlling secretions (the cause of the "death rattle"). (Option 3) Morphine is an opioid analgesic that is effective for pain treatment as well as terminal dyspnea. The client is not exhibiting these symptoms, so morphine would be inappropriate. (Option 4) Ondansetron will help the nausea and vomiting but is not very effective for treating the "death rattle." Educational objective:The "death rattle" is a noisy rattling sound with breathing commonly seen in a dying client who is unresponsive and no longer able to manage airway secretions. Anticholinergic medications such as transdermal scopolamine or atropine sublingual drops effectively treat this symptom by drying up the excess secretions.

A client's arterial blood gases (ABGs) are shown in the exhibit. The nurse would expect which finding to demonstrate that the client is compensating for the ABGs? Click on the exhibit button for additional information. Laboratory results pH 7.25 PO2 79 mm Hg (10.5 kPa) PaCO2 35 mm Hg (4.66 kPa) HCO3. 12 mEq/L (12 mmol/L) a. Decrease in bicarbonate reabsorption b. Decrease in RR c. Increase in bicarbonate reabsorption d. Increase in RR

The client's ABGs have low pH consistent with acidosis. If it is a primary respiratory acidosis, pCO2 would be higher. If it is metabolic acidosis, bicarbonate would be lower. Because this client has low pH coupled with low bicarbonate, the most likely diagnosis is primary metabolic acidosis. Respiratory alkalosis is the body's natural compensation for metabolic acidosis. Respiratory alkalosis is achieved by blowing more CO2 off from the system through rapid breathing. (Option 1) Decreased bicarbonate reabsorption would produce metabolic acidosis; this would occur as a compensation for primary respiratory alkalosis (decreased pCO2 and high pH). (Option 2) When the respiratory rate is decreased, pCO2 would increase, creating a respiratory acidosis; this would occur in response to a primary metabolic alkalosis. (Option 3) Increased bicarbonate reabsorption would produce metabolic alkalosis; this would occur as a compensation for primary respiratory acidosis (increased pCO2 and low pH). Educational objective:Respiratory alkalosis is the body's natural compensation for metabolic acidosis. It is achieved by blowing more CO2 off from the system through rapid breathing.

A nurse is caring for a child who is receiving oxygen at 2 L/min by nasal cannula and observes the current oxygen saturation and pulse plethysmographic waveform on the pulse oximeter. Which intervention should be the nurse's initial action? Click the exhibit button for additional information. a. Auscultate the child's lung fields b. Have the child take slow, deep breaths c. Increase the oxygen flow rate to 3 L/min d. Verify the position and integrity of the finger probe

The first action of the nursing process is assessment. The nurse should first evaluate the accuracy of the reading by evaluating the pulse plethysmographic waveform. Waveforms that are irregular or erratic may contain artifact caused by a loose, misapplied, or damaged pulse oximeter or by client movement (Option 4). After ensuring that the probe has been properly applied and positioned to provide an accurate reading, the nurse should perform a thorough physical assessment and intervene as appropriate. (Option 1) Auscultation of the lungs would be the next appropriate action to perform if pulse oximetry readings are deemed accurate. (Options 2 and 3) If the reading is deemed accurate after further assessment, the nurse should assist the client into the high Fowler position and encourage slow, deep breaths to promote ventilation. The nurse should increase the oxygen flow rate and notify the health care provider if the client's oxygen saturation does not improve with nursing interventions. Educational objective:When a low oxygen saturation with apparent artifact in the pulse plethysmographic waveform is observed, the nurse should discern the accuracy of the reading to prevent unnecessary treatment. If the pulse oximeter reading is accurate, the nurse should perform a thorough physical assessment and intervene as appropriate.

The nurse cares for a client who returns from the operating room after a tracheostomy tube placement procedure. Which of the following is the nurse's priority when caring for a client with a new tracheostomy? a. Changing the inner cannula within the first 8 hours to help prevent mucus plugs b. Checking the tightness of ties and adjusting if necessary, allowing 1 finger to fit under these ties c. Deflating and reinflating the cuff every 4 hours to prevent mucosal damage d. Performing frequent mouth care every 2 hours to prevent infection

The immediate postoperative priority goal for a client with a new tracheostomy is to prevent accidental dislodgement of the tube and loss of the airway. If dislodgement occurs during the first postoperative week, reinsertion of the tube is difficult as it takes the tract about 1 week to heal. For this reason, dislodgement is a medical emergency. The priority nursing action is to ensure the tube is placed securely by checking the tightness of ties and allowing for 1 finger to fit under these ties. (Option 1) Changing of the inner cannula and tracheostomy ties is not usually performed until 24 hours after insertion; this is due to the risk of dislodgement with an immature tract. However, the dressing can be changed if it becomes wet or soiled. Suctioning can be performed to remove mucus and maintain the airway. (Option 3) The cuff is kept inflated to prevent aspiration from secretions and postoperative bleeding. Cuffs are not regularly deflated and re-inflated. The respiratory therapist should monitor the amount of air in the cuff several times a day to prevent excessive pressure and mucosal tissue damage. (Option 4) Frequent mouth care to help prevent stomal and pulmonary infection is important in a client with an artificial airway, but it is not the priority action immediately following tracheostomy. Educational objective:The immediate postoperative priority goal for a client with a newly placed tracheostomy is to prevent accidental dislodgement of the tube and loss of the airway.

The triage nurse is assessing an unvaccinated 4-month-old infant for fever, irritability, and open-mouthed drooling. After the infant is successfully treated for epiglottitis, the parents wonder how this could have been avoided. Which response by the nurse would be most appropriate? a. It's impossible to know for sure what could have caused this episode b. Most cases of epiglottitis are preventable by standard immunizations c. We are still waiting for the formal report from the microbioligy laboratory d. There is nothing you could have done; the important thing is that your child is safe now.

The majority of cases of epiglottitis are caused by Haemophilus influenza type B (HiB), which is covered under the standard vaccinations given during the 2- and 4-month visits. Epiglottitis is rarely seen in vaccinated children. (Option 1) This statement is technically true, but it is not helpful to the parents and misses a critical teaching moment for them. (Option 3) It is reasonable to attribute the cause of the infant's epiglottitis to missing the vaccinations for Haemophilus influenza type B. (Option 4) This statement is both unhelpful and inaccurate as the child is still at risk for further preventable illness. Educational objective:Cases of epiglottitis are preventable, and parents should always be educated on the risks of foregoing vaccinations for their children.

The office nurse instructs a client newly diagnosed with asthma about the use of the peak flow meter to evaluate airflow. Which statement made by the client indicates an understanding of the nurse's teaching? a. I will exhale as quickly and forcibly as possible through the mouthpiece of the device to obtain a peak flow reading b. I will move the indicator to the desired reading on the numbered scale before using the device c. I will record my personal best reading, which is the average of 3 consecutive peak flow readings d. I will remember to use the device after taking my fluticasone MDI.

The peak flow meter is a hand-held device used to measure peak expiratory flow rate (PEFR) and is most helpful for clients with moderate to severe asthma. Exhaling as quickly and forcibly as possible through the mouthpiece of the device evaluates the degree of airway narrowing by measuring the volume of air that can be exhaled in one breath. Use of the device permits self-management and provides information to guide and evaluate treatment. (Option 2) The client moves the indicator on the numbered scale to 0 or to the lowest number on the scale before using the device. (Option 3) The personal best reading is the highest peak flow reading the client can attain, usually over a 2-week period, when asthma is in good control. (Option 4) The peak flow meter is used after a short-acting bronchodilator rescue MDI to evaluate response, not after a corticosteroid MDI. Educational objective:The peak flow meter is used to measure PEFR and is most helpful for clients with moderate to severe asthma. A reading is obtained by exhaling as quickly and forcibly as possible through the mouthpiece of the device.

The nurse is teaching a 9-year-old child with asthma how to use a metered-dose inhaler (MDI). Place the instructions in the appropriate order. All options must be used. 1. Deliver one puff of medication into the spacer 2. Take a slow deep breath and hold for 10 seconds 3. Shake MDI and attach it to the spacer 4. Place lips tightly around the mouthpiece 5. Rinse mouth with water 6. Exhale completely

The proper method of delivering a dose via MDI includes the following steps: First shake MDI and attach it to the spacer. Exhale completely to optimize inhalation of the medication. Place lips tightly around the mouth piece. Deliver a single puff of medication into spacer. Take a slow, deep breath and hold it for 10 seconds to allow for effective medication distribution. After the dose, rinse mouth with water to remove any left-over medication from oral mucous membranes. Spit out the water to ensure no medication is swallowed. Educational objective:Any child under age 12 should use a spacer with the MDI to ensure the entire dose is inhaled appropriately.

The nurse is evaluating how well a client with chronic obstructive pulmonary disease understands the discharge teaching. Which statements made by the client indicate an understanding of the pursed-lip breathing technique? Select all that apply. a. I exhale for 2 seconds through pursed lips b. I exhale for 4 seconds through pursed lips c. I inhale for 2 seconds through my mouth d. I inhale for 2 seconds through my nose, keeping my mouth closed. e. I inhale for 4 seconds through my nose, keeping my mouth closed.

The pursed-lip breathing technique helps to decrease shortness of breath by preventing airway collapse, promoting carbon dioxide elimination, and reducing air trapping in clients with chronic obstructive pulmonary disease (COPD). Clients with COPD are taught to use this technique when experiencing dyspnea as it increases ventilation and decreases work of breathing. Regular practice (eg, 5-10 minutes 4 times daily) enables the client to do pursed lip breathing when short of breath, without conscious effect. Clients are taught the following steps: Relax the neck and shoulders Inhale for 2 seconds through the nose with the mouth closed (Option 4) Exhale for 4 seconds through pursed lips. If unable to exhale for this long, exhale twice as long as inhaling (Option 2). (Option 1) Exhalation through pursed lips is done for 4 seconds, not 2 seconds, or twice as long as inhalation. (Option 3) Inhalation is done through the nose, not the mouth. (Option 5) Inhalation is done for 2 seconds, not 4 seconds. Educational objective:The pursed-lip breathing technique helps to decrease shortness of breath by preventing airway collapse, promoting carbon dioxide elimination, and reducing air trapping in clients with chronic obstructive pulmonary disease. Clients are taught to relax the shoulders and neck, inhale through the nose for 2 seconds with the mouth closed, and exhale through pursed lips for 4 seconds (or twice as long as inhalation).

The medical surgical nurse cares for a client who had a mediastinal tumor removed 2 days ago and reports difficulty breathing. The client becomes confused and restless, and respirations are 30/min. What is the nurse's next action? a. Administer a dose of prescribed prn anti-anxiety medication b. Call the HCP who performed the sx c. Call the Rapid Response Team d. Place the client in a left lateral recovery position

The rapid response team (RRT) consists of a group of health care providers who bring critical care expertise to the bedside of clients demonstrating early signs of deterioration such as dyspnea, confusion, and restlessness. This team differs from the "Code" team that is called when a client stops breathing or goes into cardiac arrest. Any health care worker can call the RRT. (Option 1) The client's restlessness and confusion are likely secondary to low oxygenation. Anxiety will cause hyperventilation, which will only exacerbate the situation. However, administering anti-anxiety medication is not the priority over obtaining help quickly. In addition, the client's oxygenation could deteriorate depending on the prescribed anti-anxiety medication, which could depress respirations. (Option 2) The health care provider who performed the surgery must be notified of the client's deteriorating condition; however, this should be done after calling the RRT. Stabilizing the client is the priority. (Option 4) The recovery position is used as a first aid measure for an unconscious client who is still breathing. The client is placed on the left or right side in a three-fourths prone position with the top leg flexed. This position maintains the airway and ensures that the client does not choke on vomit. Educational objective:When a client is demonstrating clinical deterioration, the nurse's priority is to prevent full respiratory or cardiac arrest by calling the rapid response team.

An elderly client with a history of stable chronic obstructive pulmonary disease, alcohol abuse, and cirrhosis has a serum theophylline level of 25.8 mcg/mL (143 µmol/L). Which clinical manifestation associated with theophylline toxicity should worry the nurse most? a. Alterations in color vision b. Gum (gingival) hypertrophy c. Hyperthermia d. Seizure activity

Theophylline has narrow therapeutic index and plasma concentrations >20 mcg/mL (111 µmol/L) are associated with theophylline drug toxicity. Toxicity can be acute or chronic. Conditions associated with chronic toxicity include advanced age (>60), drug interactions (eg, alcohol, macrolide and quinolone antibiotics), and liver disease. Acute toxicity is associated with intentional or accidental overdose. Symptoms of toxicity usually manifest as central nervous system stimulation (eg, headache, insomnia, seizures), gastrointestinal disturbances (eg, nausea, vomiting), and cardiac toxicity (eg, arrhythmia). (Option 1) Alteration in color perception and visual changes are commonly seen with digoxin toxicity. (Option 2) Gum hypertrophy is seen with phenytoin toxicity. (Option 3) Hyperthermia and tinnitus are often seen with aspirin overdose. Educational objective:Theophylline plasma concentrations >20 mcg/mL (111 µmol/L) are associated with theophylline drug toxicity. Seizures (central nervous system stimulation) and cardiac arrhythmias are the most serious and lethal consequences.

The nurse is teaching a client with advanced chronic obstructive pulmonary disease who was prescribed oral theophylline. Which client statement indicates that additional teaching is required? a. I need to avoid caffeinated products b. I need to get my blood drug levels checked periodically c. I need to report anorexia and sleeplessness d. I take Cimetidine rather than omeprazole for heartburn.

Theophylline is a bronchodilator with a low therapeutic index and a narrow therapeutic range (10-20 mcg/mL). The serum level should be monitored frequently to avoid severe adverse effects. Toxicity is likely to occur at levels >20 mcg/mL. Individual titration is based on peak serum theophylline levels, so it is necessary to draw a blood level 30 minutes after dosing. Theophylline can cause seizures and life-threatening arrhythmias. Toxicity is usually due to intentional overdose or concurrent intake of medications that increase serum theophylline levels. Cimetidine and ciprofloxacin can dramatically increase serum theophylline levels (>80%). Therefore, they should not be used in these clients. (Option 1) Caffeinated products (eg, coffee, cola, chocolate) should be avoided as they would intensify the adverse effects (eg, tachycardia, insomnia, restlessness) of theophylline. (Option 2) The best way to prevent toxicity is to monitor drug levels periodically and adjust the dose. (Option 3) The signs of toxicity that should be reported are anorexia, nausea, vomiting, restlessness, and insomnia. Educational objective:Theophylline can cause seizures and life-threatening arrhythmias due to its narrow therapeutic range (10-20 mcg/mL). The dose is adjusted based on peak drug levels, obtained 30 minutes after the dose is given. Clients should avoid caffeinated products and medications that increase serum theophylline levels (eg, cimetidine, ciprofloxacin).

The home care nurse is making an initial visit to a client just discharged after admission for severe exacerbation of chronic obstructive pulmonary disease (COPD). The nurse observes wall-to-wall stacks of old newspapers and magazines in every room, with pathways that just allow passage from one room to another. What is the priority nursing action? a. Call the mobile community mental health crisis unit b. Contact a service to remove newspapers and magazines c. Reconcile the client's discharge medications d. Teach the safe use of oxygen

This client exhibits signs of hoarding disorder, an anxiety disorder defined by the Diagnostic and Statistical Manual of Mental Disorders, 5th Edition (American Psychiatric Association) as persistent difficulty with discarding or parting with possessions, even those of little value. Clients with hoarding disorder will typically accumulate items such as clothing, food, boxes, bags, newspapers, and magazines. These items commonly fill up and clutter their living areas and can create environmental and fire hazards. The client will most likely experience severe anxiety if the items are removed. Clients with hoarding disorder may never seek mental health services or come to the attention of a mental health professional. Their own behavior usually does not concern them, although it may cause great distress in family members or friends. The treatment for the client with severe COPD will include home oxygen therapy. The priority nursing action is to ensure the safety of the client when using oxygen in an environment that is already at high risk for a hazardous event. (Option 1) Referral to mental health services is an appropriate intervention but is not the priority nursing action. (Option 2) The nurse cannot intervene in this manner without the consent of the client. Removing the newspapers and magazines could cause the client to experience severe anxiety. (Option 3) Reconciling the client's discharge medications is appropriate but is not the priority nursing action. Educational objective:Teaching the safe use of home oxygen is the priority nursing intervention for a client with hoarding disorder who lives in an environment at high risk for fire due to the accumulation of newspapers and magazines. Hoarding disorder is the persistent difficulty with discarding possessions, no matter their value. Removal of the items will cause the client to experience severe anxiety.

An elderly client is admitted with chronic obstructive pulmonary disease (COPD) exacerbation. Pulse oximetry is 84% on room air. The client is restless, has expiratory wheezing and a productive cough, and is using his accessory muscles to breathe. Which prescription should the nurse question? a. Albuterol 2.5 mg by nebulizer b. Methylprednisolone 125 mg IV now and every 6 hours c. Morphine 2 mg IV now and may repeat every 2 hours d. Oxygen at 2L/min via n/c

This client has exacerbation (wheezing) of chronic obstructive pulmonary disease (COPD). Restlessness is an early, subtle sign of hypoxemia. Although morphine can be used to decrease restlessness and slow the respiratory rate to decrease oxygen demand, it can depress respirations. Morphine and other medications (eg, benzodiazepines) that can depress the respiratory center should not be used in clients with COPD exacerbation as they can further worsen CO2 retention. The oxygen and medications should increase saturation and decrease restlessness, pulse, and respiratory rate. (Option 1) Albuterol is a beta agonist and is appropriate for the immediate relief of bronchoconstriction because of its rapid, short action. (Option 2) Methylprednisolone (Solu-Medrol) is a corticosteroid and is appropriate for decreasing inflammation of the lungs during an acute exacerbation of COPD. (Option 4) The initiation of low flow, low concentration oxygen at 2 L/min by nasal cannula is appropriate in a client with COPD. It is best to start oxygen at a lower concentration and titrate upward if the saturation does not reach 90% within 20-30 minutes as many clients with COPD rely on their hypoxemic drive to breathe. Educational objective:Morphine and other medications (eg, benzodiazepines) that can depress the respiratory center should not be used in clients with COPD exacerbation as they can further worsen CO2 retention.

A 2-year-old child is brought to the emergency department for a severe sore throat and fever of 102.9 F (39.4 C). The nurse notes that the child is drooling with distressed respirations and inspiratory stridor. What action should the nurse take first? a. Assess an accurate temperature with a rectal thermometer b. Directly examine the throat for the presence of exudates c. Obtain intravenous access for anticipated steroid administration d. Position the child in tripod position on the parent's lap

This is a classic description of epiglottitis (supraglottitis). It is an inflammation by bacteria of the tissues surrounding the epiglottis, a long, narrow structure that closes off the glottis during swallowing. Edema can develop rapidly (as quickly as a few minutes) and obstruct the airway by occluding the trachea. There has been a 10-fold decrease in its incidence due to the widespread use of the Hib (Haemophilus influenzae type B) vaccine. The classic symptoms include a high-grade fever with toxic appearance, severe sore throat, and the 4 Ds—dysphonia (muffled voice), dysphagia (difficulty swallowing), drooling, and distressed respiratory effort. The tripod position opens the airway and helps air flow. The child should be allowed to assume a position of comfort (usually sitting rather than lying down). The priority nursing response is to protect the airway. (Option 1) No invasive procedure should be done that could cause the child to cry until the airway is secure. Knowing the temperature is not a priority. (Options 2 and 3) When drooling is present, the airway becomes the primary concern. No visual inspection, invasive procedure, or anxiety-provoking activity should be done until the airway is secure due to the risk of laryngospasm and respiratory arrest. Educational objective:Children with potential epiglottitis should be allowed a position of comfort without any invasive or anxiety-provoking procedures (eg, phlebotomy, pharyngeal examination, epiglottal cultures) until the airway is secure with intubation or a surgical airway.

A client with bronchial asthma and sinusitis has increased wheezing and decreased peak flow readings. During the admission interview, the nurse reconciles the medications and notes that which of the following over-the-counter medications taken by the client could be contributing to increased asthma symptoms? a. Guaifenesin 600 mg orally twice a day as needed b. Ibuprofen 400 mg PO every 6 hours for pain as needed c. Loratidine 1 tab PO every day as needed d. Vitamin D 2,000 units PO every day

Two groups of commonly used drugs, nonsteroidal anti-inflammatory drugs and beta-adrenergic antagonists (beta blockers), have the potential to cause problems for clients with asthma. Ibuprofen (Motrin) and aspirin are common over-the-counter anti-inflammatory drugs that are effective in relieving pain, discomfort, and fever. About 10%-20% of asthmatics are sensitive to these medications and can experience severe bronchospasm after ingestion. This is prevalent in clients with nasal polyposis. (Option 1) Guaifenesin (Mucinex) is an expectorant used to facilitate mobilization of mucus and should not have the potential to exacerbate asthma or cause an attack. (Option 3) Loratadine (Claritin) is an antihistamine and should not have the potential to exacerbate asthma or cause an attack. (Option 4) Vitamin D is used to help maintain bone density and should not have the potential to exacerbate asthma or cause an attack. Educational objective:Ibuprofen and aspirin are common over-the-counter anti-inflammatory drugs that can cause bronchospasm in some clients with asthma.

The nurse is caring for a client who has been receiving mechanical ventilation (MV) for 4 days. During multidisciplinary morning rounds, the health care provider questions the development of a ventilator-associated pneumonia (VAP). Which of the following manifestations does the nurse assess as the best indicator of VAP? a. Blood tinged sputum b. Positive blood cultures c. Positive, purulent sputum culture d. Rhonchi and crackles

VAP is the second most common health care-associated infection (HAI) in the United States and is associated with increased mortality, hospital cost, and length of stay. Because it is a nosocomial infection, signs and symptoms associated with VAP usually present within ≥2-3 days after initiation of mechanical ventilation (MV). Characteristic clinical manifestations of VAP include purulent sputum, positive sputum culture, leukocytosis (12,000 mm3), elevated temperature (>100.4 F [38 C]), and new or progressive pulmonary infiltrates suggestive of pneumonia on chest x-ray. (Option 1) Blood-tinged sputum may occur but is not the best indicator of VAP. (Option 2) Positive blood cultures may identify the microorganism causing the infection but are not the best indicator of VAP. Positive blood cultures could be from another source of infection. (Option 4) Rhonchi and crackles are adventitious lung sounds associated with pneumonia but can be present in pulmonary edema or just from increased mucous secretions. They are not the best indicator of VAP. Educational objective:VAP is an HAI that usually occurs within ≥2-3 days after the initiation of mechanical ventilation. Characteristic manifestations of VAP include purulent secretions, positive sputum culture, leukocytosis, elevated temperature, and new or progressive pulmonary infiltrates on chest x-ray.

Which pediatric respiratory presentation in the emergency department is a priority for nursing care? a. Client with an acute asthma exacerbation but no wheezing b. Client with bronchiolitis with low-grade fever and wheezing c. Client with runny nose with seal like barking cough d. Cystic fibrosis client with fever and yellow sputum

When an acute asthma exacerbation occurs, the child has rapid, labored respirations using accessory muscles. The child often appears tired due to the ongoing effort. In the case of severe obstruction (from airway narrowing as a result of bronchial constriction, airway swelling, and copious mucus), wheezing/breath sounds are not heard due to lack of airflow. This "silent chest" is an ominous sign and an emergency priority. In this situation, the onset of wheezing will be an improvement as it shows that air is now moving in the lungs. (Option 2) Bronchiolitis is associated with the respiratory syncytial virus (RSV). Cell debris clumps and clogs the airways. Air can get in but has difficulty getting out. Mild symptoms include low-grade fever, wheezing, tachypnea, and poor feeding; severe infections have more serious distress, including signs of hypoxia. Treatment is supportive. This child should be isolated and will receive supportive care, but the child with no air movement/wheezing is a priority. (Option 3) Croup or laryngotracheobronchitis is a viral inflammation and edema of the epiglottis and larynx. Symptoms include runny nose, tachypnea, inspiratory stridor, and a seal-like barking cough. The child will be treated with medications and oxygen (if needed). The child is still moving air. (Option 4) In clients with cystic fibrosis, fever with yellow or green sputum can be indicative of an infection. The child will receive antibiotics but is not a priority as there are no signs of respiratory distress. Educational objective:Inability to hear any breath sounds or wheezing in an acute asthma client ("silent chest") is an ominous sign and requires emergency intervention.

The nurse is caring for a client admitted with incomplete fractures of right ribs 5-7. The nurse notes shallow respirations, and the client reports deep pain on inspiration. What is the priority at this time? a. Administer prescribed IV morphine b. Facilitate hourly client use of IS c. Instruct client on gently splinting injury during coughing d. Notify the HCP immediately

a. Administer prescribed IV morphine Rib fractures are often the result of blunt thoracic trauma (eg, motor vehicle collision). In the absence of significant internal injuries (eg, pneumothorax, pulmonary contusion, spleen laceration), interventions focus on pain management and pulmonary hygiene techniques (eg, coughing, deep breathing, incentive spirometry). Breaths may become shallow as the client experiences pain with inspiration, which can result in a buildup of secretions, atelectasis, and pneumonia. The nurse should ensure adequate pain control prior to encouraging pulmonary hygiene techniques (Option 1). (Options 2 and 3) Interventions focused on removing secretions to improve gas exchange (eg, ambulation, coughing, incentive spirometry) are appropriate after the client's pain is controlled. (Option 4) Rib fractures are very painful. Shallow breathing and reports of pain on inspiration are expected findings that do not require immediate notification of the health care provider. Educational objective:Client management for rib fractures focuses on pain control followed by pulmonary hygiene techniques (eg, coughing, deep breathing, incentive spirometry). Without adequate pain control, breathing can become shallow, which may lead to buildup of secretions, atelectasis, and pneumonia

A nursing diagnosis of "ineffective airway clearance related to pain" is identified for a client who had open abdominal surgery 2 days ago. Which intervention should the nurse implement first? a. Administer prescribed analgesic medication for incisional pain b. Encourage use of IS every 2 hours while awake c. Offer an additional pillow to splint the incision while coughing d. Promote increased oral fluid intake

a. Administer prescribed analgesic medication for incisional pain Postoperative clients are at risk for atelectasis and possibly for pneumonia following surgery as a result of retained secretions. Effective coughing is essential to prevent these complications. The nurse can promote many client actions that will facilitate effective coughing. These include splinting the incision while coughing, changing position every 1-2 hours, ambulating early, using an incentive spirometer, and hydrating adequately to thin the secretions. However, all of these interventions are less effective if the client is in pain. The nurse should instruct the client to request pain medication before the pain becomes intense. Pain relief should be addressed prior to implementing coughing exercises and ambulation. (Options 2, 3, and 4) These are appropriate interventions but will be more effective if pain is managed first. Educational objective:The nurse should ensure that the postoperative client has effective pain relief before performing coughing exercises.

The client with malignant left pleural effusion undergoes a thoracentesis and 900 mL of excess pleural fluid is removed. Which of these manifestations, if noted on the post-procedure assessment, should the nurse report to the health care provider immediately? a. Asymmetrical chest expansion and decresed breath sounds on the left b. BP 100/65 c. Client complains of 6/10 pain at the needle insertion site d. Respiratory rate 24/min, pulse ox 94% on O2 2L/min

a. Asymmetrical chest expansion and decresed breath sounds on the left Complications from insertion of the needle and removal of large amounts of fluid include iatrogenic pneumothorax, hemothorax, pulmonary edema, and infection. After the procedure, the nurse assesses for pain and difficulty breathing; monitors vital signs and oxygen saturation; and observes for changes in respiratory rate and depth, symmetry of chest expansion, and breath sounds. If any abnormalities are noted, a post-procedure chest x-ray is obtained. Decreased chest expansion with inspiration and breath sounds on the affected side, tachypnea, tracheal deviation to the opposite side, and hyperresonance (air) on the affected side are manifestations of a pneumothorax. These should be reported immediately. (Option 2) Hypotension, pulmonary edema, and tachycardia can occur as the result of removal of large amounts of pleural fluid (>1.5 L). This client's blood pressure is adequate (mean arterial pressure 77 mm Hg), and the nurse should continue to monitor. However, this blood pressure does not need to be reported immediately. (Option 3) Mild to moderate pain is common after the procedure. It does not need to be reported immediately. (Option 4) Difficulty breathing, tachypnea, and hypoxemia are pulmonary complications that can occur after thoracentesis. Saturation (94%) and respiratory rate (24/min) are adequate and do not need to be reported immediately. Educational objective:Complications of thoracentesis include iatrogenic pneumothorax, hemothorax, and infection. Post-procedure, the nurse assesses for pain and difficulty breathing; monitors vital signs and oxygen saturation; and observes for changes in respiratory rate and depth, symmetry of chest expansion, and breath sounds.

GOOD The nurse is caring for a client with a chest tube to evacuate a hemopneumothorax after a motor vehicle accident. The drainage has been consistently 25-50 mL/hr for the majority of the shift. However, over the past 2 hours there has been no drainage. Which actions should the nurse take? Select all that apply. a. Auscultate breath sounds b. Increase amount of suction c. Instruct client to cough and deep breathe d. Milk the chest tube e. Reposition the client

a. Auscultate breath sounds c. Instruct client to cough and deep breath e. Reposition the client When chest drainage stops abruptly, the nurse must perform assessments and interventions to ascertain if this is an expected finding. Auscultating breath sounds (Option 1) helps the nurse detect whether breath sounds are audible in all lung fields, potentially indicating that the lung has re-expanded and there is no more drainage. Other interventions to facilitate drainage include having the client cough and deep breathe (Option 3) and repositioning the client (Option 5). If a client has been in one position for a prolonged period, drainage may accumulate and a position change may facilitate improved drainage. (Option 2) A change in suction level should be performed only after obtaining a health care provider (HCP) prescription. The nurse should perform the assessment of breath sounds, coughing and deep breathing, and client repositioning before notifying the HCP about a change in suction level. In general, suction above 20 cm H2O is not indicated. (Option 4) Milking chest tubes to maintain patency is performed only if prescribed. It is generally contraindicated due to potential tissue damage from highly increased pressure changes in the pleural space. Educational objective:The nurse should assess breath sounds, encourage coughing and deep breathing, and reposition the client who has a decrease in chest tube drainage.

A client with chronic kidney disease has a large pleural effusion. What findings characteristic of a pleural effusion does the nurse expect? Select all that apply. a. Chest pain during inhalation b. Diminished breath sounds c. Dyspnea d. Hyperresonance on percussion e. Wheezing

a. Chest pain during inhalation b. Diminished breath sounds c. Dyspnea A pleural effusion is an abnormal collection of fluid (>15 mL) in the pleural space that prevents the lung from expanding fully, resulting in decreased lung volume, atelectasis, and ineffective gas exchange. It is usually secondary to another disease (eg, heart failure, pneumonia, nephrotic syndrome). Pleural effusions are diagnosed by chest x-ray or CT scan. Thoracentesis can be performed to remove fluid from the pleural space and resolve symptoms. Clients commonly report dyspnea with a nonproductive cough, as well as pleural chest pain with respirations (Options 1 and 3). On assessment, clients have diminished breath sounds, dullness to percussion, decreased tactile fremitus, and decreased movement over the affected lung (Option 2). (Option 4) Fluid outside the lung interrupts the transmission of sound, resulting in decreased fremitus and dullness with percussion in pleural effusion. Percussion is hyperresonant in clients with pneumothorax. (Option 5) Wheezing indicates an obstructive process (eg, asthma, chronic obstructive pulmonary disease) and is not typical in pleural effusion. Educational objective:A pleural effusion is an abnormal collection of fluid (>15 mL) in the pleural space that prevents the lung from expanding fully, resulting in decreased lung volume, atelectasis, and ineffective gas exchange. Clients report dyspnea and pain with respirations and have diminished breath sounds with dullness to percussion over the affected area.

The nurse is preparing medication for 4 clients on a respiratory medical-surgical unit. Which situation would prompt the nurse to clarify the prescribed treatment with the health care provider? a. Client with bronchospasm who is due to receive nebulized acetylcysteine b. Client with COPD due to receive PO prednisone c. Client with Cystic Fibrosis who is due to receive PO pancrelipase with breakfast d. Client with suspected bacterial PNA due to receive IV levoflaxacin

a. Client with bronchospasm who is due to receive nebulized acetylcysteine Acetylcysteine (Mucomyst) may be given via nebulizer to help loosen and liquefy respiratory secretions to more easily clear them from the airway. Inhaled acetylcysteine may be used for clients with cystic fibrosis or other respiratory conditions with thick bronchial mucus. Acetylcysteine has no therapeutic effect on airway smooth muscle as it works primarily on secretions and has been shown to cause and/or worsen bronchospasm. Nurses caring for clients with reactive airway diseases (eg, asthma) prescribed acetylcysteine should clarify the prescription with the health care provider (Option 1). (Option 2) Chronic obstructive pulmonary disease (COPD) is a respiratory illness in which excess mucus, inflamed bronchioles, and easily collapsible airways trap air within the alveoli. Oral corticosteroids (eg, prednisone) may be used to reduce airway inflammation and improve ventilation in clients with acute COPD exacerbation. (Option 3) Cystic fibrosis is a genetic condition that causes dehydration and thickening of mucus in the respiratory, gastrointestinal, and genitourinary systems. Thick mucus within the pancreas impairs the release of digestive enzymes (eg, lipase), requiring supplementation to improve digestion and prevent malnutrition in clients with CF. (Option 4) Levofloxacin (Levaquin) is a broad-spectrum antibiotic that may be used to treat respiratory tract infections, such as bacterial pneumonia. Educational objective:Acetylcysteine is a medication that can be inhaled to help loosen thick respiratory secretions. Nurses caring for clients with reactive airway diseases (eg, asthma) who are prescribed acetylcysteine should clarify the prescription with the health care provider as it may cause and/or worsen bronchospasm.

A client was medicated with intravenous morphine 2 mg 2 hours ago to relieve moderate abdominal pain after appendectomy. The client becomes lethargic but arouses easily to verbal and tactile stimuli, and is oriented to time, place, and person. The pulse oximeter reading has dropped from 99% to 89% on room air. Which oxygen delivery device is the most appropriate for the nurse to apply? a. Nasal cannula b. Non-rebreather mask c. Simple face mask d. Venturi mask

a. Nasal cannula The nasal cannula is the most appropriate oxygen delivery device to apply at this time because it is comfortable, used for the short term, inexpensive, and permits the client to eat and drink fluids. It can supply adequate oxygen concentrations of up to 44%. This client is most likely hypoventilating as a result of the opioid medication. The client is alert and oriented and able to follow directions. Because pain relief is effective according to the pain scale, the client should be able to breathe deeply through the nose, and the hypoxemia should reverse rapidly. (Option 2) The non-rebreather mask is used in emergencies, delivers high concentrations of oxygen (up to 90%-95%), requires a tight face seal, and is restrictive and uncomfortable. (Option 3) The simple face mask delivers a higher concentration of oxygen (40%-60%), is more uncomfortable and restrictive, must be removed to eat or drink, and is not appropriate at this time. It can be used if hypoxemia does not resolve. (Option 4) The Venturi mask is a more expensive device used to deliver a guaranteed oxygen concentration to clients with unstable chronic obstructive pulmonary disease. These clients cannot tolerate changes in oxygen concentration. Educational objective:The nasal cannula is an inexpensive, comfortable, low-flow oxygen delivery device capable of delivering oxygen concentrations of up to 44%. It can be used in the short term in responsive postoperative clients to treat hypoventilation and reverse hypoxemia effectively.

GOOD A nurse is caring for an elderly client who had a colectomy for removal of cancer 2 days ago. The client is becoming increasingly restless. He has been given intravenous morphine every 2 hours for severe pain. Respirations are 28/min and shallow. Which arterial blood gas (ABG) results best indicate that the client is in acute respiratory failure (ARF) and needs immediate intervention? a. PaO2 49; PaCO2 60 b. PaO2 64; PaCO2 45 c. PaO2 70; PaCO2 30 d. PaO2 86; PaCO2 25

a. PaO2 49; PaCO2 60 ARF is defined as inadequate gas exchange that is intrapulmonary (pneumonia, pulmonary embolism) or extrapulmonary (head injury, opioid overdose) in origin. Respiratory failure associated with an alteration in O2 transfer or absorption is type I hypoxemic failure (eg, acute respiratory distress syndrome, pulmonary edema, shock). Respiratory failure associated with carbon dioxide (CO2) retention is type II hypercapnic, or ventilatory failure (eg, chronic obstructive pulmonary disease, myasthenia gravis, flail chest). ARF is a potential complication of major surgical procedures, especially those involving the thorax and abdomen, as in this client. ABG values that indicate the presence of ARF are PaO2 ≤60 mm Hg (8.0 kPa) or PaCO2 ≥50 mm Hg (6.67 kPa). ARF occurs quickly over time (minutes to hours), and so there is no physiologic compensation and pH is ≤7.30. Immediate intervention with high O2 concentrations is indicated, and noninvasive or invasive, positive-pressure mechanical ventilation may be necessary. (Option 2) PaO2 64 mm Hg (8.5 kPa) indicates hypoxemia, and PaCO2 45 mm Hg (6.0 kPa) is within the normal range, but results do not meet the criteria for ARF. (Option 3) PaO2 70 mm Hg (9.3 kPa) indicates hypoxemia, and PaCO2 30 mm Hg (4.0 kPa) indicates increased ventilation and an alkalotic state, but results do not meet the criteria for ARF. (Option 4) PaO2 86 mm Hg (11.5 kPa) is within normal range, and PaCO2 25 mm Hg (3.33 kPa) indicates increased ventilation and an alkalotic state, but results do not meet the criteria for ARF. Educational objective:Type I hypoxemic failure is associated with an alteration in O2 transfer (eg, acute respiratory distress syndrome, pulmonary edema, shock). Type II hypercapnic, or ventilatory, failure is associated with CO2 retention (eg, chronic obstructive pulmonary disease, myasthenia gravis, flail chest). ABG values that indicate the presence of ARF are PaO2 ≤60 mm Hg (8.0 kPa), PaCO2 ≥50 mm Hg (6.67 kPa), and pH ≤7.30.

A self-employed auto mechanic is diagnosed with carbon monoxide poisoning. Admission vital signs are blood pressure 90/42 mm Hg, pulse 84/min, respirations 24/min, and oxygen saturation 94% on room air. What is the nurse's priority action? a. Administer 5 mg inhaled albuterol treatment to decrease inflammatory bronchoconstriction b. Administer 100% oxygen using a nonrebreather mask with flow rate of 15L/min c. Administer methyprednisolone to decrease lung inflammation from toxic inhalant d. Titrate oxygen to maintain pulse oximeter saturation of >95%

b. Administer 100% oxygen using a nonrebreather mask with flow rate of 15L/min The purpose of hemoglobin (Hgb) is to pick up oxygen in the lungs and deliver it to the tissues. It must be able to pick up oxygen and release it in the right places. Carbon monoxide (CO) has a much stronger bond to Hgb than oxygen does. Consequently, CO displaces oxygen from Hgb, causing hypoxia that is not reflected by a pulse oximeter reading. The nurse's primary action is to administer highly concentrated (100%) oxygen using a nonrebreather mask at 15 L/min in order to reverse this displacement of oxygen. (Option 1) Albuterol is not a priority action as bronchoconstriction is not a consequence of CO poisoning. (Option 3) Administration of corticosteroids is not a priority/primary action as direct inflammation of the lungs is not an underlying cause for hypoxemia and hypoxia associated with CO poisoning. (Option 4) When all available Hgb binding sites are occupied (oxyhemoglobin or carboxyhemoglobin), saturation (SaO2) is 100%. The conventional pulse oximeter cannot differentiate carboxyhemoglobin from oxyhemoglobin as both absorb the oximeter's red and infrared light wavelengths. Consequently, the pulse oximeter reading may be adequate (>90%), but severe hypoxemia and hypoxia may be present. Alternate methods of CO saturation measurement (eg, multiple wavelength CO pulse oximeter, spectrographic blood gas analysis) are recommended. Educational objective:The conventional pulse oximeter is not effective in identifying hypoxia in CO poisoning; diagnosis requires co-oximetry of a blood gas sample. The priority action is to administer 100% oxygen using a nonrebreather mask to treat hypoxia and help eliminate CO.

A client with asthma was recently prescribed fluticasone/salmeterol. After the client has received instructions about this medication, which statement would require further teaching by the nurse? a. After taking this medication, I will rinse my mouth with water b. At the first sign of an asthma attack, I will take his medication c. I have been smoking for 12 years, but I just quit a month ago d. I received the pneumococcal vaccine about a month ago.

b. At the first sign of an asthma attack, I will take his medication Fluticasone/salmeterol (Advair) is a combination drug containing a corticosteroid (fluticasone) and a bronchodilator (salmeterol). Salmeterol is a long-acting inhaled β2-adrenergic agonist that promotes relaxation of the bronchial smooth muscles over 12 hours. Fluticasone decreases inflammation. This medication is used as part of the treatment plan for prevention and long-term control of asthma. Client instructions include: After inhalation, rinse the mouth with water without swallowing to reduce the risk of oral/esophageal candidiasis Avoid smoking and using tobacco products Receive the pneumococcal and influenza vaccines if there is a risk for infection (Option 2) Fluticasone/salmeterol is not a rescue inhaler and does not treat acute exacerbations of asthma. The client should always have a rescue inhaler (eg, albuterol [short-acting β2-adrenergic agonist] or ipratropium [Atrovent]) for sudden changes in breathing and call 911 if the rescue inhaler does not relieve the breathing problem. Educational objective:Fluticasone/salmeterol (Advair) is a long-acting inhaled β2-adrenergic agonist combination drug containing a corticosteroid (fluticasone) and a bronchodilator (salmeterol). It is used for long-term control of asthma but not for acute attacks.

The nurse cares for a client with a pulmonary embolism. Which of the following clinical manifestations would the nurse anticipate? Select all that apply. a. Bradycardia b. Chest pain c. Dyspnea d. Hypoxemia e. Tachypnea f. Tracheal deviation

b. Chest pain c. Dyspnea d. Hypoxemia e. Tachypnea Pulmonary embolism (PE) is a potentially life-threatening medical emergency occurring when a blood clot, fat or air embolus, or tissue (eg, tumor) travels via the venous system into the pulmonary circulation and obstructs blood flow into the lung. This prevents deoxygenated blood from reaching the alveoli, which leads to hypoxemia due to impaired gas exchange and cardiac strain due to congested blood flow in the pulmonary arteries. Clinical manifestations of PE range from mild (eg, anxiety, cough) to severe (eg, heart failure, sudden death). However, many clients initially have mild, nonspecific symptoms that are often misdiagnosed and inadequately managed, greatly increasing the likelihood of progression to shock and/or cardiac arrest. Clinical manifestations of PE include: Pleuritic chest pain (ie, sharp lung pain while inhaling) (Option 2) Dyspnea and hypoxemia (Options 3 and 4) Tachypnea and cough (eg, dry or productive cough with bloody sputum) (Option 5) Tachycardia Unilateral leg swelling, erythema, or tenderness related to deep vein thrombosis (Option 1) Tachycardia, rather than bradycardia, is expected with PE because the heart attempts to compensate for hypoxemia, right ventricular overfilling, and decreased left ventricular cardiac output. (Option 6) Tracheal deviation is a sign of tension pneumothorax (not PE), which occurs when pressure on the side of the collapsed lung pushes organs toward the unaffected lung. Educational objective:Pulmonary embolism is a potentially life-threatening medical emergency occurring when a pulmonary artery is obstructed. Common clinical manifestations include pleuritic chest pain, dyspnea, hypoxemia, tachypnea, cough, tachycardia, and unilateral leg swelling.

A client with an asthma exacerbation has been using her albuterol rescue inhaler 10-12 times a day because she cannot take a full breath. What possible side effects of albuterol does the nurse anticipate the client will report? Select all that apply. a. Constipation b. Difficulty sleeping c. Hives with pruritus d. Palpitations e. Tremor

b. Difficulty sleeping d. Palpitations e. Tremor Albuterol is a short-term beta-adrenergic agonist used as a rescue inhaler to treat reversible airway obstruction associated with asthma. Dosing in an acute asthma exacerbation should not exceed 2-4 puffs every 20 minutes x 3. If albuterol is not effective, an inhaled corticosteroid is indicated to treat the inflammatory component of the disease. Albuterol is a sympathomimetic drug. Expected side effects mimic manifestations related to stimulation of the sympathetic nervous system, and commonly include insomnia, nausea and vomiting, palpitations (from tachycardia), and mild tremor. (Option 1) Constipation is not a common side effect of inhaled beta-agonist drugs. (Option 3) Hives can occur as a sign of an allergic reaction and are not a common anticipated side effect of an inhaled beta-agonist drug. Educational objective:Albuterol is a short-term beta-agonist rescue drug used to control symptoms of airway obstruction and promote bronchodilation. It is a sympathomimetic drug; common expected side effects include insomnia, nausea and vomiting, palpitations (tachycardia), and mild tremor.

The nurse develops a care plan for a critically ill client with acute respiratory distress syndrome (ARDS) who is on a mechanical ventilator. What is the priority nursing diagnosis (ND)? a. Imbalanced nutrition b. Impaired gas exchange c. Impaired tissue integrity d. Risk for infection

b. Impaired gas exchange ARDS involves damage to the alveolar-capillary membrane, the blood-gas barrier across which oxygen diffuses into the alveoli. When the membrane is damaged, the alveoli collapse and fluid leaks into the alveolar space and impairs gas exchange. Impaired gas exchange related to alveolar-capillary changes and ventilation-perfusion imbalance is the priority ND for a client with ARDS. (Option 1) Imbalanced nutrition (less than body requirements) related to increased metabolic needs and inability to ingest foods due to endotracheal intubation, is an appropriate ND for the client with ARDS. However, it does not pose the greatest threat to survival and is not the priority ND. (Option 3) Impaired tissue (integumentary) related to altered circulation, immobility, and nutritional deficits is an appropriate ND for the client with ARDS. However, it does not pose the greatest threat to survival and is not the priority ND. (Option 4) Risk for infection related to the presence of an endotracheal tube, frequent suctioning, intravenous devices, and indwelling catheters is an appropriate ND for the client with ARDS. However, it does not pose the greatest threat to survival and is not the priority ND. Educational objective:ARDS involves damage to the alveolar-capillary membrane, resulting in fluid leakage into the alveolar space. Impaired gas exchange related to alveolar-capillary changes and ventilation-perfusion imbalance is an appropriate ND for a client with ARDS.

A client with a severe asthma exacerbation following influenza infection is transferred to the intensive care unit due to rapidly deteriorating respiratory status. Which clinical manifestations support the nurse's assessment of impending respiratory failure? Select all that apply. a. Arterial pH 7.50 b. PaCO2 55 c. PaO2 58 d. Paradoxical breathing e. Restlessness and drowsiness

b. PaCO2 55 c. PaO2 58 Acute severe asthma exacerbations (status asthmaticus) occur when severe airway obstruction and lung hyperinflation (air trapping) persist despite aggressive treatment with bronchodilators and corticosteroid therapy. Clinical manifestations indicating impending respiratory failure include: PaCO2 ≥45 mm Hg (6.0 kPa): Indicates hypercapnia and hypoventilation resulting from fatigue and labored breathing. As initial tachypnea subsides and respiratory rate returns to normal, PaCO2 rises and respiratory acidosis develops (Option 2). PaO2 ≤60 mm Hg (8.0 kPa): Indicates hypoxemia resulting from increased work of breathing, decreased gas exchange (hyperinflation and air trapping), and inability of the lungs to meet the body's oxygen demand (Option 3) Paradoxical breathing (ie, abnormal inward movement of the chest on inspiration and outward movement on expiration): Indicates diaphragm muscle fatigue and use of respiratory accessory muscles (Option 4) Mental status changes (eg, restlessness, confusion, lethargy, drowsiness): Sensitive indicators of hypoxemia and hypoxia (Option 5) Absence of wheezing and silent chest (ie, no sound of air movement on auscultation): Ominous signs indicating severe hyperinflation and air trapping in the lungs Single-word dyspnea: Inability to speak >1 word before pausing to breathe due to shortness of breath (Option 1) Normal arterial pH is 7.35-7.45. A pH of 7.50 indicates alkalosis, which could be respiratory or metabolic. Clients with respiratory failure have respiratory acidosis (low pH and elevated pCO2). Educational objective:Clinical manifestations indicating impending respiratory failure in clients with asthma include hypercapnia, hypoxemia, paradoxical breathing, and mental status changes.

In the emergency department, a pediatric client is placed on mechanical ventilation by means of an endotracheal tube. Several hours later, the nurse enters the room and finds the client in respiratory distress. It is most important for the nurse to take which of these actions? a. Assess the client for intercostal retractions b. Assess the client's BP in both arms c. Auscultate the client's lung sounds d. Observe the color of the client's fingernail beds

c. Auscultate the client's lung sounds A client experiencing respiratory distress while receiving mechanical ventilation should be assessed for proper ventilation first. The nurse needs to determine if the mechanical ventilation equipment is still properly placed in the trachea. An endotracheal tube (ET) can become displaced with movement. By assessing the client's lung sounds, the nurse can quickly determine if ET placement has been compromised (Option 3). Airway is the priority for this client. By auscultating the client's lung sounds, the nurse can determine if the client has an open airway. (Option 1) This is an assessment of the client's breathing, which is not the priority at this time. (Option 2) This is an assessment of the client's circulation, which is not the priority at this time. (Option 4) This is an assessment of the client's circulation, which is not the priority at this time. Educational objective:Clients with respiratory distress should be assessed for a patent airway first. The nurse should assess the client's airway to determine if it is present or needs to be established.

After receiving a change-of-shift report, the nurse should assess which client first? a. Client recently admitted with a positive tuberculin skin test who reports hemoptysis b. Client who had abdominal surgery yesterday and refuses incentive spirometry c. Client who has just returned to the floor after undergoing the bronchoscopy d. Client with a chest tube drainage system with air bubbles in the suction control chamber

c. Client who has just returned to the floor after undergoing the bronchoscopy The client who just underwent a bronchoscopy is at risk for becoming unstable. During a bronchoscopy, the larynx, trachea, and bronchi are visualized using an endoscope while the client is under sedation. Respiratory status, airway patency, vital signs, and sedation level should be assessed immediately upon return from the procedure and at regular intervals until the client becomes stable. The client must be kept NPO until alert with a positive gag reflex. (Option 1) Clients with a positive tuberculin skin test and hemoptysis may have tuberculosis. They will need a chest x-ray and sputum culture to confirm the diagnosis and may be initiated on an antitubercular medication regimen. However, this client does not require immediate attention. (Option 2) The nurse should address any factors that may cause a client to refuse incentive spirometry (eg, pain). This client may require education on the importance of incentive spirometry (eg, prevention of atelectasis and pneumonia) or administration of PRN analgesics to encourage participation in deep-breathing exercises. However, this client does not require immediate attention. (Option 4) Air bubbles in the suction control chamber of a chest tube drainage system would be a normal finding. Air bubbles in the water seal chamber may indicate a leak and would require immediate intervention. Educational objective:Clients returning from procedures performed under sedation should receive priority assessment of vital signs and respiratory status.

Several children are brought to the emergency room after a boating accident in which they were thrown into the water. The children are now 6 hours post admission to the clinical observation unit. Which client should the nurse evaluate first? a. Client who did not require CPR but now has a new oxygen requirement of 2L via nasal cannula to maintain a saturation of 95% b. Client who did not require CPR but was coughing on arrival to the hospital and is now crying inconsolably and asking for the mother c. Client who received CPR for 2 minutes on the scene and whose respiratory rate has now dropped from 61/mn to 18/min d. Client who was briefly submerged in water and received rescue breaths on the scene and is now irritable and refusing food and drink

c. Client who received CPR for 2 minutes on the scene and whose respiratory rate has now dropped from 61/mn to 18/min Clients with morbidity related to immersion in water are described as having submersion injury. Even if an individual was submerged for a very brief time, it is possible that water may have been aspirated, which can lead to respiratory compromise. Observation for at least 6 hours is recommended as the majority of significant respiratory problems will manifest in this time period. A marked decrease in respiratory rate or increased work of breathing may indicate respiratory fatigue, and immediate intervention is needed (Option 3). Impending respiratory failure is the immediate priority. (Option 1) A new oxygen requirement is an important symptom; however, this child has good oxygen saturation with the nasal cannula and is therefore not the immediate priority. (Option 2) This child who is coughing and emotionally distressed should be seen and comforted by the nurse but is not the priority. (Option 4) Irritability can be an early sign of hypoxia in a toddler. This child should be assessed promptly but is not the immediate priority. Educational objective:Clients who have sustained submersion injury should be evaluated immediately and observed for at least 6 hours for new or worsening respiratory failure. Changes in respiratory pattern or rate, oxygen saturation, and level of consciousness can signal impending respiratory failure, which can be life threatening.

A nurse in an urgent care center triages multiple clients. Which client should the nurse assess first? a. Client who reports nosebleed that has not resolved after holding pressure for 1 hour b. Client who reports sinus congestion with thick nasal drainage and severe facial pain c. Client with a sore throat who reports difficulty in opening mouth and swallowing d. Client with seasonal allergies who reports new onset of unilateral ear pain and pressure

c. Client with a sore throat who reports difficulty in opening mouth and swallowing Peritonsillar, or retropharyngeal, abscess is a serious complication that can result from tonsillitis or pharyngitis. The presenting features of peritonsillar abscess, in addition to fever, include a "hot potato" (muffled) voice, trismus (inability to open the mouth), pooling of saliva (drooling), and deviation of the uvula to one side. The abscess can progress to life-threatening airway obstruction (eg, dysphagia, stridor, restlessness). The nurse should immediately assess the client with symptoms of peritonsillar abscess and monitor for signs of airway obstruction (Option 3). (Option 1) A client with epistaxis (ie, nosebleed) that does not resolve with external pressure will require further hemostatic interventions, such as cauterization or nasal packing (eg, gauze, nasal tampon, balloon catheter). This client should be assessed after the client with signs of impending airway obstruction. (Option 2) Symptoms of acute sinusitis include severe facial pain, nasal congestion with purulent nasal drainage, and fever. In most cases, the etiology is viral but can be complicated by secondary bacterial infection. This client likely requires antibiotics and supportive care but is not the priority. (Option 4) Acute otitis media (ie, infection of the middle ear) may develop secondary to rhinitis (eg, common cold, seasonal allergies) due to inflammation of the Eustachian tube. This client with otitis media will likely require antibiotics and pain management but is not the priority. Educational objective:Peritonsillar abscess is an emergent complication of tonsillitis that can lead to life-threatening airway obstruction. Symptoms of peritonsillar abscess include fever, trismus (inability to open the mouth), drooling, muffled voice, and deviation of uvula to one side.

The nurse responds to the call light of a client with chronic obstructive pulmonary disease (COPD) who says, "I can't breathe." The client seems to be having difficulty breathing and is nervous and tremulous. Vital signs are stable, oxygen saturation is 92% on 2 L, and there are clear breath sounds bilaterally. Which intervention would be most appropriate at this time? a. Administer albuterol nebulizer b. Assist the client in identifying the trigger and ways to avoid it c. Coach the client through the controlled breathing exercises d. Continue to monitor oxygen saturation

c. Coach the client through the controlled breathing exercises Anxiety is an emotional reaction to a perceived threat. For the client with COPD, the fear of having difficulty breathing can actually trigger difficulty breathing, which worsens as the client's anxiety increases. This client is stable, with no obvious cause of shortness of breath. The nurse should intervene by calmly coaching the client through breathing exercises, which will promote relaxation and help alleviate the anxiety that is causing the client to feel short of breath. (Option 1) The client's lung sounds are clear bilaterally and so albuterol, a bronchodilator used for wheezing, will not be helpful. Its action as an adrenergic agonist may cause tachycardia and tremulousness and actually worsen the client's anxiety. (Option 2) Trigger avoidance and problem solving are appropriate strategies for long-term control of anxiety and shortness of breath. However, these are not appropriate at this time as the client has acute symptoms that need to be controlled. (Option 4) This client has normal oxygen saturation. Constant monitoring is not likely to alleviate the symptoms unless the client is reassured by this knowledge. However, the client's anxiety may actually be worsened by worrying about the saturation results and the alarms that are likely to be triggered by monitoring. Educational objective:Anxiety is common in clients with COPD and can contribute to difficulty breathing. In the client with acute shortness of breath and normal assessment findings, appropriate interventions are controlled breathing and relaxation.

The nurse is caring for a client with advanced heart failure on an inpatient hospice unit. The client is having trouble breathing. Which comfort intervention should the nurse implement first? a. Administer PRN albuterol by nebulizer b. Administer prn IV furosemide c. Elevate the HOB d. Give prn sublingual morphine

c. Elevate the HOB Dyspnea, difficulty breathing, is a common symptom in the client with advanced heart failure who is on hospice. The most common cause of dyspnea is fluid overload, and elevating the head of the bed is often an effective intervention that can be implemented quickly and easily. (Option 1) Albuterol is a bronchodilator and is unlikely to relieve feelings of dyspnea caused by fluid overload. (Options 2 and 4) Additional interventions for dyspnea in heart failure include the use of IV furosemide to promote diuresis to treat the fluid overload. The use of prn morphine can alleviate the sensation of dyspnea, but this would not be the first intervention; it should be used in combination with diuresis if the dyspnea is associated with fluid overload. Educational objective:The client with advanced heart failure on hospice is likely to have dyspnea associated with fluid overload. The first intervention should be to elevate the head of the bed and then assess for fluid overload, which would be treated with IV diuretics. Morphine can alleviate dyspnea associated with heart failure, but it should be used in combination with other nonpharmacologic and pharmacologic interventions.

The nurse assesses a child who has been treated for an acute asthma exacerbation. Which client assessment is the best indicator that treatment has been effective? a. Episodes of spasmodic coughing have decreased b. No wheezes are audible on chest auscultation c. Oxygen saturation has increased from 88% to 93% d. Peak expiratory flow rate has dropped from 212 L/min to 127L/min

c. Oxygen saturation has increased from 88% to 93% Asthma is a chronic condition characterized by inflammation, swelling, and narrowing of the airways in the lungs. The client having an acute attack will experience chest tightness, wheezing, uncontrollable coughing, rapid respirations, retractions, and anxiety and panic. Treatment of an acute attack can include nebulized breathing treatment with a short-acting beta-agonist medication such as albuterol, and oral or IV corticosteroids. Oxygen saturation is the best indicator of treatment effectiveness as it reflects gas exchange. (Option 1) Decreased coughing may indicate improvement, but it is more subjective than measurement of oxygen saturation. In addition, it may be a sign of client exhaustion and worsening asthma. (Option 2) The absence of wheezes may indicate resolution of the attack or progression of airway swelling to the point of little air flowing through the lungs. (Option 4) Peak expiratory flow rate, by measuring how much air a person can exhale, indicates the amount of airway obstruction. Following treatment for an acute asthma attack, an increase, not a decrease, in peak expiratory flow would be expected. Educational objective:Improvements in oxygen saturation and peak expiratory flow are the best indicators of treatment effectiveness during an acute asthma attack.

The nurse is providing care for a client with cancer of the left lung who will undergo video-assisted thoracic surgery in the morning. The client is nervous, jumpy, and short of breath. Pulse is 120/min, respirations are 30/min and shallow, and expiratory wheezing is auscultated in the left upper and lower lung posteriorly. Which of the following is the priority nursing action? a. Administered prescribed intravenous morphine 2 mg to relieve anxiety b. Page RT to administer inhaled bronchodilator nebulizer treatment c. Place HOB in Fowler's or high Fowler's position d. Stay with client and encourage client to discuss feelings about the surgery.

c. Place HOB in Fowler's or high Fowler's position Elevating the head of the bed to Fowler's or high Fowler's position is the priority nursing action to help relieve shortness of breath, facilitate oxygenation (breathing), and promote lung expansion (airway). Alternate positions to high Fowler's include the following: Orthopneic position: Sitting in a chair, on the side, or in bed leaning over the bedside table, with one or more pillows under the arms or elbows for support Tripod position: Sitting in a chair leaning forward with hands or elbows resting on the knees. Sitting upright and leaning forward pulls the scapulae apart, promotes lung expansion, and decreases the diaphragmatic pressure produced by the viscera. (Option 1) Morphine is effective in relieving anxiety and decreasing the work of breathing by slowing respirations. It can cause hypoventilation and decrease gas exchange in the lungs and is not the priority action, especially as the client's respirations are shallow. (Option 2) The cause of the wheezing could be from lung tumor or true bronchoconstriction. Paging the respiratory therapist to administer a bronchodilator nebulizer treatment to relieve wheezing is an appropriate intervention, but it is not the priority action. (Option 4) Encouraging the client to talk about the diagnosis and upcoming surgery is an appropriate intervention to help alleviate anxiety and address self-actualization needs, but is not the priority action. Educational objective:Elevating the head and chest in the Fowler's, high Fowler's, orthopneic, and tripod positions allows for maximum lung expansion and promotes oxygenation, especially in clients with dyspnea.

When an unlicensed assistive personnel (UAP) assists a client with a chest tube back to bed from the bedside commode, the plastic chest drainage unit accidently falls over and cracks. The UAP immediately reports this incident to the nurse. What is the nurse's immediate action? a. Clamp the tube close to the client's chest until a new chest drainage unit is set up b. Notify the HCP c. Place the distal end of the chest tube into a bottle of sterile saline d. Position the client on the left side

c. Place the distal end of the chest tube into a bottle of sterile saline If the chest tube disconnects from the drainage tubing without contamination, wipe the end of the chest tube with an antiseptic and immediately reconnect it. To prevent accidental disconnection of the chest tube from the tubing, secure all connections with tape or bands, according to hospital policy and procedure. If the chest tube is disconnected with contamination and cannot be immediately reattached, or if the chest drainage unit breaks, cracks, or malfunctions, submerge the distal end of the chest tube 1-2 in (2-4 cm) below the surface of a 250 mL bottle of sterile water or saline. This creates an immediate water seal and prevents air from entering into the pleural space as the new chest drainage system is established (Option 3). To be prepared for this contingency, emergency equipment should be kept at the bedside, which includes 2 chest tube clamps, a 250 mL bottle of sterile water or saline solution, and antiseptic wipes. (Option 1) Unless prescribed by the HCP, chest tube clamping time should not exceed 1 minute as it raises intrapleural pressure and can lead to a tension pneumothorax. Clamping briefly is acceptable when checking for an air leak in the system or when changing the disposable collection unit. (Option 2) It is not necessary to notify the HCP when replacing a chest drainage system unless the client develops respiratory distress. (Option 4) Positioning the client on the left side is appropriate if a central line is inadvertently pulled out so that any air that may have been sucked in will rise to the right atrium. It is not an appropriate intervention for a chest tube disconnection or crack or malfunction in a chest drainage unit. Educational objective:If a chest tube disconnects from the chest drainage system and cannot be reattached quickly, or if a chest drainage unit cracks or malfunctions, submerge the distal end of the chest tube 1-2 in (2-4 cm) below the surface of a 250 mL bottle of sterile water or saline solution.

The nurse is caring for a client with asthma exacerbation. Blood pressure is 146/86 mm Hg, pulse is 110/min, and respirations are 32/min. The respiratory therapist administers nebulized albuterol as prescribed. One hour after the treatment, the nurse assesses which finding that indicates the drug is producing the therapeutic effect? a. Constricted pupils b. Heart rate of 120/min c. Respirations of 24/min d. Tremor

c. Respirations of 24/min The most frequent side effects are tremor, tachycardia, restlessness, and hypokalemia. Albuterol (Proventil) is a short-acting inhaled beta-2 agonist used to control airway obstruction caused by chronic obstructive pulmonary disease, asthma, or bronchitis. It also is used to prevent exercise-induced asthma. The therapeutic effect is relaxation of the smooth muscles of the airways, which results in immediate bronchodilation. Bronchodilation decreases airway resistance, facilitates mucus drainage (expectorates mucus plugs), decreases the work of breathing, and increases oxygenation. As a result of these actions, the respiratory rate will decrease and peak flow will be increased (if tested). However, short-acting beta-2 agonists are associated with the following side effects (not therapeutic effects): tremor (most frequent), tachycardia and palpitations, restlessness, and hypokalemia. These side effects are due to the oral deposition of medication (subsequent systemic absorption) and can be reduced with the use of a spacer or chamber device. (Option 1) The presence of constricted pupils is neither a side effect nor therapeutic effect of the drug. Constricted pupils are often seen with opioid medications (eg, morphine, oxycodone). Educational objective:Albuterol (Proventil) is a short-acting beta-2 agonist that produces immediate bronchodilation by relaxing smooth muscles. Bronchodilation decreases airway resistance, facilitates mucus drainage, decreases the work of breathing, and increases oxygenation. Peak flow will improve. The most frequent side effects are tremor, tachycardia, restlessness, and hypokalemia.

The nurse is caring for a client with a chest tube that was placed 2 hours ago for a pneumothorax. Where would the nurse expect gentle, continuous bubbling? a. Air leak monitor b. Collection chamber c. Suction control chamber d. Water seal chamber

c. Suction control chamber controls suction to the chest drainage system; continuous, gentle bubbling indicates that the suction level is appropriate. The amount of suction is controlled by the amount of water in the chamber and not by wall suction. Increasing the amount of wall suction would cause vigorous bubbling but does not increase suction to the client as excess suction is drawn out through the vent of the suction control chamber. Vigorous bubbling would increase water evaporation and therefore decrease the negative pressure applied to the system. The nurse should check the water level and add sterile water, if necessary, to maintain the prescribed level. (Option 1) The air leak monitor (Section C) is part of the water seal chamber. Continuous or intermittent bubbling seen here indicates the presence of an air leak. (Option 2) The collection chamber (Section D) is where drainage from the client will accumulate. The nurse will assess amount and color of the fluid and record as output. (Option 4) The water seal chamber contains water, which prevents air from flowing into the client. Up and down movement of fluid (tidaling) in Section B would be seen with inspiration and expiration and indicates normal functioning of the system. This will gradually reduce in intensity as the lung reexpands. Educational objective:The nurse should observe gentle, continuous bubbling in the suction control chamber. This indicates patency and the appropriate level of suction being applied to the drainage system.

The nurse is assisting the health care provider (HCP) with a client's chest tube removal. Just as the HCP prepares to pull the chest tube, what instructions should the nurse give the client? a. Breathe as you normally would b. Inhale and exhale slowly c. Take a breath in, hold it, and bear down d. Take a rapid shallow breaths, similar to panting

c. Take a breath in, hold it, and bear down Chest tubes are indicated to drain air or fluid from the pleural space and reestablish negative pressure, which allows for proper lung expansion. When the lung has reexpanded or fluid drainage is no longer needed, the chest tube can be discontinued. The client should be given an analgesic 30-60 minutes prior to the procedure. A suture removal kit, petroleum gauze, and occlusive dressing supplies will be needed. The client should be instructed to take a deep breath, hold it, and bear down (Valsalva maneuver) while the tube is being removed. This will prevent air from being pulled back into the pleural space and possibly causing a pneumothorax. A post-procedure chest x-ray must be performed to ensure there is no reaccumulation of air or fluid in the pleural space. (Options 1 and 2) Breathing slowly or normally during the procedure may cause the client to inhale during the removal, pulling air back into the pleural space. (Option 4) Rapid shallow breaths increase the chance of inhaling during removal and pulling air into the pleural space, causing recollapse of the lung. Educational objective:During chest tube removal, the client should be instructed to take a deep breath, hold it, and bear down (Valsalva maneuver) to prevent air from reentering the pleural space and possibly causing a pneumothorax. The site is covered with a sterile airtight petroleum jelly gauze dressing. A post-procedure chest x-ray is needed.

The nurse receives the handoff of care report on four clients. Which client should the nurse see first? a. Client reporting incisional pain 8/10 with RR 25/min who had a right pneumonectomy 12 hours ago b. Client with a left pleural effusion who has crackles, absent breath sounds in the left base, and an SpO2 of 94% RA c. Client with a temp of 100.4 F and a RR - 12 who had a small bowel resection 1 day ago d. Client with PNA who has a temp of 97.6 and O2 sat - 93% on 4L oxygen and is becoming restless

d. Client with PNA who has a temp of 97.6 and O2 sat - 93% on 4L oxygen and is becoming restless Acute respiratory failure (ARF) is a life-threatening impairment of the lungs' ability to oxygenate blood and excrete carbon dioxide (CO2). ARF may occur from exacerbation of chronic (eg, chronic obstructive pulmonary disease, asthma) or acute (eg, pneumonia, pulmonary edema) illnesses. Nurses assessing for signs of ARF should consider both respiratory and neurological manifestations. Altered mental status (eg, confusion, agitation, somnolence) is a common and often overlooked symptom that may occur because of the brain's sensitivity to inadequate oxygenation and alterations in acid-base balance from retained CO2(Option 4). Additional signs and symptoms may include paresthesias, dyspnea, tachypnea, and hypoxemia. (Option 1) Clients recovering from recent pneumonectomy (ie, surgical removal of part or all of the lung) often experience considerable pain, which may cause respiratory distress if not adequately controlled. A client with tachypnea and severe pain should be seen promptly but only after addressing potential ARF. (Option 2) Crackles, absent or diminished breath sounds over the affected lobe, and slightly decreased oxygen saturation are expected findings in pleural effusion, in which fluid collects in the space surrounding the lung. (Option 3) Low-grade fever may occur following surgery (due to the release of inflammatory cytokines) or from postoperative atelectasis. The client should be encouraged to ambulate and deep-breathe. Educational objective:Acute respiratory failure is a life-threatening impairment of lung function that inhibits gas exchange. Common symptoms include altered mental status (eg, confusion, agitation, somnolence), paresthesias, dyspnea, tachypnea, and hypoxemia, all of which should be addressed immediately.

Based on the lung assessment information included in the hand-off report, which client should the nurse assess first? a. Client 1-day postoperative abdominal surgery who has fine inspiratory crackles at the lung bases b. Client with chronic bronchitis who has rhonchi in the anterior and posterior chest c. Client with right sided pleural effusion who has decreased breath sounds at the right lung base d. Client with severe acute pancreatitis who has inspiratory crackles at the lung bases.

d. Client with severe acute pancreatitis who has inspiratory crackles at the lung bases. Clients with acute pancreatitis can develop respiratory complications including pleural effusions, atelectasis, and acute respiratory distress syndrome (ARDS). These complications are often due to activated pancreatic enzymes and cytokines that are released from the pancreas into the circulation and cause focal or systemic inflammation. ARDS is the most severe form of these complications and can rapidly progress to respiratory failure within a few hours. Therefore, the presence of inspiratory crackles in this client could indicate early ARDS and needs to be assessed further for progression. (Option 1) Fine crackles are a series of distinct, discontinuous, and high-pitched snapping sounds usually heard on inspiration. The sound originates as small atelectatic bronchioles quickly reinflate and can be expected in clients who have undergone abdominal surgery due to shallow breathing related to pain. Although the presence of fine crackles requires treatment (eg, ambulation, deep breathing), this is not the priority assessment. (Option 2) Rhonchi are continuous, low-pitched wheezes usually heard on expiration that sound like moaning or snoring. The sound originates from air moving through large airways (bronchi) filled with mucus secretions and are expected in clients with chronic bronchitis. Although they require treatment (eg, medication, mobilization of secretions), this is not the priority assessment. (Option 3) The lung under the pleural effusion is compressed, and the breath sounds are decreased/absent if auscultated over the area; this is an expected finding. Until the pleural effusion is treated with diuretics or thoracentesis, these findings will remain unchanged. Educational objective:Clients with acute pancreatitis are at high risk for developing acute respiratory distress syndrome.

A client presents to the emergency department with a stab wound to the chest. The nurse assesses tachycardia, tachypnea, and a sucking sound coming from the wound. Which of the following actions is priority? a. Administer prescribed IV fluids b. Apply supplemental oxygen via nonrebreather mask c. Assist the HCP to prepare for chest tube insertion d. Cover the wound with petroleum gauze taped on three sides

d. Cover the wound with petroleum gauze taped on three sides In a traumatic, or "open," pneumothorax, air rushes in through the wound with each inspiration, creating a sucking sound, and fills the pleural space. The lungs cannot expand, so the client develops respiratory distress and air hunger. Tachycardia and hypotension result from impaired venous return, as the heart and great vessels shift with each breath. A tension pneumothorax may also develop if air cannot escape the pleural space. The priority action in this medical emergency is to apply a sterile occlusive dressing (eg, petroleum gauze) taped on three sides, preventing inward air flow while allowing air to escape the pleural space. (Option 1) This client's tachycardia and hypotension are likely related to pneumothorax and should improve once the pneumothorax is resolved; administering fluids alone would not help if the pneumothorax continues to worsen. Fluids are given to treat blood loss hypotension, but this should not be the first step in this case. (Option 2) Supplemental oxygen should be applied as needed after covering the wound. If possible, correcting the underlying cause is always a priority over treating manifestations. (Option 3) After covering the wound, chest tube placement is usually performed to evacuate air and blood from the pleural cavity. The client may need more than one chest tube to evacuate both air (placed higher) and fluid or blood (placed lower). Educational objective:A sucking chest wound indicates a traumatic, or "open," pneumothorax and is a medical emergency. Respiratory distress results from inability to expand the lung. The priority action is to apply a sterile occlusive dressing (eg, petroleum gauze) taped on three sides.

A client is admitted with a pulmonary embolus. The nurse assesses restlessness, one-word dyspnea and shortness of breath with activity, tachycardia, pleuritic chest pain, and severe anxiety. Arterial blood gases indicate respiratory alkalosis and hypoxemia. When initiating the care plan, the nurse should choose which nursing diagnosis as the highest priority? a. Activity intolerance r/t imbalance between oxygen supply and demand b. Acute pain r/t inspiration and inflammation of pleura c. Anxiety r/t fear of the unknown, chest pain and dyspnea d. Impaired gas exchange r/t ventilation-perfusion imbalance

d. Impaired gas exchange r/t ventilation-perfusion imbalance Pulmonary embolism (PE) is usually caused by a dislodged thrombus that travels through the pulmonary circulation, becomes lodged in a pulmonary vessel, and causes an obstruction to blood flow in the lung. The nursing diagnosis of impaired gas exchange involves an alteration in the normal exchange of oxygen and carbon dioxide at the alveolar-capillary membrane, resulting in inadequate oxygenation and hypoxemia (respiratory alkalosis, pO2 <80 mm Hg, restlessness, dyspnea, and tachycardia). Impaired gas exchange related to a ventilation-perfusion (V/Q) imbalance is the highest priority nursing diagnosis. It addresses the most basic physiologic need—oxygen. Clients will not survive without adequate oxygenation. (Options 1, 2, and 3) Activity intolerance, acute pain, and anxiety elicit autonomic responses (exertional discomfort, dyspnea, tachycardia) and are all appropriate nursing diagnoses. However, none are the highest priority or pose the greatest threat to survival. Educational objective:Activity intolerance, anxiety, acute pain, and impaired gas exchange are all appropriate nursing diagnoses to include in the plan of care for a client with PE. The highest priority nursing diagnosis is the one that poses the greatest threat to the client's survival.

The nurse is assessing a client an hour after a left lung lobectomy. The client is awake, alert, and oriented, and reports pain of 6 on a 0-10 scale. Pulse oximetry is 92% on 4 L oxygen via nasal cannula. The chest tube is set to continuous water seal suction at -20 cm H2O. The collection chamber has accumulated 320 mL of frank red drainage in the last hour. What is the priority nursing action? a. Clamp the chest tube immediately b. Increase oxygen to 6 L via n/c c. Medicate the client for pain and document the findings d. Notify the health care provider immediately

d. Notify the health care provider immediately Following lung surgery, a chest tube is inserted into the pleural space to create a negative vacuum to re-inflate the lung and prevent air from re-entering the space. A client with a chest tube should be assessed for signs of air/fluid in the chest (eg, diminished breath sounds), excessive drainage (>100 mL/hr), pain, and infection at the drainage site. The collection chamber should be inspected every hour for the first 8 hours following surgery, then every 8 hours until it is removed. Excess drainage of frank red blood is indicative of hemorrhage and must be managed immediately. The priority action is to contact the health care provider for further management. (Option 1) Clamping the chest tube prevents air or fluid from leaving the pleural space, which may cause a reciprocal tension pneumothorax. The chest tube is clamped only a few hours prior to removal, momentarily to check for an air leak, or if the drainage apparatus needs to be changed. (Option 2) Although a pulse oximetry of 92% is low, this is an expected finding following lung surgery. (Option 3) Pain following surgery is a concern and the client will require medication; however, hemorrhage is the priority. Educational objective:A client with a chest tube should be assessed for signs of air/fluid in the chest (eg, diminished breath sounds), excessive drainage (>100 mL/hr), pain, and infection at the drainage site. Excess drainage of frank red blood is indicative of hemorrhage and must be managed immediately.

The nurse receives an obese client in the postanesthesia care unit who underwent a procedure under general anesthesia. The nurse notes an oxygen saturation of 88%. Which is the most appropriate initial intervention? a. Assess pupillary response b. Auscultate lung sounds c. Inform anesthesia professional d. Perform head tilt and chin lift

d. Perform head tilt and chin lift Head tilt and chin lift is a maneuver used to open the airway. The tongue may fall back and occlude the airway due to muscular flaccidity after general anesthesia. Manifestations associated with airway obstruction include snoring, use of accessory muscles, decreased oxygen saturations, and cyanosis. (Option 1) Constricted pupils can help identify opioid overdose. However, this should not be assessed before opening the airway. (Option 2) Auscultation of lung sounds should be done for every client as part of the postoperative assessment. However, the initial goal is to return the oxygen saturation level to normal (95%-100%). Hypoxia in an obese postoperative client who received general anesthesia is most likely due to airway obstruction. (Option 3) The anesthesia professional may need to be informed, but methods to restore the oxygen saturation level should be tried first. The anesthesia professional may then want to assess the sedation level of the client and prescribe a reversal agent. Educational objective:Postoperative client care after general anesthesia requires careful monitoring for hypoxia. One of the first nursing interventions is the head tilt and chin lift to open an occluded airway.

A 64-year-old hospitalized client with chronic obstructive pulmonary disease exacerbation has increased lethargy and confusion. The client's pulse oximetry is 88% on 2 liters of oxygen. Arterial blood gas analysis shows a pH of 7.25, PO2 of 60 mm Hg (8.0 kPa), and PCO2 of 80 mm Hg (10.6 kPa). Which of the following should the nurse implement first? a. Administer PRN nebulizer treatment b. Administer scheduled dose of methylprednisolone IV c. Increase client's oxygen to 4 liters d. Place client on the bilevel positive airway pressure machine (BIPAP)

d. Place client on the bilevel positive airway pressure machine (BIPAP) An elevated carbon dioxide (CO2) level (normal: 35-45 mm Hg [4.7-6.0 kPa]) is usually an indicator of hypercapneic respiratory failure. The bilevel positive airway pressure (BIPAP) machine will provide positive pressure oxygen and expel CO2 from the lungs. This client is already showing signs of lethargy and confusion, which is usually a late indicator of respiratory decline. Therefore, the nurse's priority should be to get the client on the BIPAP machine as soon as possible. (Option 1) Nebulizer treatments are commonly part of the treatment plan for a client with chronic obstructive pulmonary disease (COPD). However, these do not take priority when the client has CO2 retention and is deteriorating. If mental status worsens further (due to continued CO2 retention), the client will need intubation. Many BIPAP machines are able to deliver nebulizer treatment while providing positive pressurized oxygen. (Option 2) Steroid therapy is a common pharmaceutical intervention for COPD exacerbation, but it does not take priority over BIPAP in this deteriorating client. In addition, steroids take hours to days to have an effect. (Option 3) In a client with an elevated CO2 level and a history of COPD, the nurse should not increase the oxygen level as this could cause an increase in CO2 retention, resulting in further respiratory failure. Educational objective:BIPAP therapy is an effective treatment to decrease CO2 levels in clients with hypercapnic respiratory failure


Related study sets

Chapter 11 - The Diversity of Prokaryotic Organisms

View Set

Congenital and genetic disorders

View Set

Chapter 10: Taxation of Life Insurance and Annuities - Premiums and Proceeds

View Set

Effects of Recreational Drugs Exam 4 | Textbook Portion

View Set

PrepU Chapter 45 Nursing Care of the Child With an Alteration in Tissue Integrity/Integumentary Disorder

View Set

Government - Chapter 5, Chapter 6 - Voting and Elections - Government, Chapter 7- Voting and Elections- Government, Chapter 8: The media - government

View Set